Chapter 7: CV

Ace your homework & exams now with Quizwiz!

Duration of Amiodarone (after hypothyroidism)

6 months

A 49-year-old man with obesity and a history of DM, HTN, and gouty arthritis has bilateral calf pain with his daily walk. He describes the pain as "crampy and achy" that is relieved when he sits down to rest. He has a 30 pack-year history of smoking but quit 6 months ago when he was given a diagnosis of diabetes. On physical examination, his feet are cool to the touch, and his dorsalis pedis and posterior tibial pulses are diminished. His blood pressure is 149/82 mm Hg, and laboratory results are A1C 8.5% and LDL 135 mg/dL. His home drugs include metformin 1000 mg twice daily, gabapentin 100 mg three times daily, amlodipine 5 mg daily, and colchicine as needed. Which would best evaluate PAD in this patient? A. ABI. B. No evaluation required. C. Toe-brachial index (TBI). D. Treadmill exercise test.

A

A 51-year-old female patient with type 2 diabetes asks whether she should take an aspirin daily. She has a family history of premature CVD but is otherwise healthy. Which information is most needed to make an appropriate recommendation for this patient according to the USPSTF recommendations for use? A. SBP, HTN treatment status, race, TC, HDL, smoking status. B. SBP, HTN treatment status, TC, HDL, smoking status. C. SBP, HTN treatment status, TC, LDL, smoking status. D. SBP, TC, HDL, DBP, smoking status.

A

Which best depicts C.A.'s blood pressure goal? A.Less than 130/80 mm Hg. B. Less than 140/90 mm Hg. C. Less than 150/90 mm Hg. D. Less than 140/80 mm Hg.

A

[ASCVD] A 62-year-old man (70 kg) with an unknown medical history presents to the ED with the chief concern of Cx pain that woke him from sleep and radiates to his jaw. An ECG reveals STEMI in leads V4-V6. His blood pressure is 142/62 mm Hg, and heart rate is 64 beats/minute. His Sao2 is 95% on room air. Cardiac markers have been obtained, and the first troponin result was positive. Preparations are under way to take the patient to the cardiac catheterization laboratory. One hour has elapsed since his first medical contact with triage. He reports taking only amlodipine 10 mg at home for blood pressure. Which treatment strategy is most appropriate for this patient at this time? A. Primary percutaneous coronary intervention (PCI) within 90 minutes. B. PCI within 120 minutes. C. Early invasive approach with reteplase 10 units intravenous push x 2. D. Tenecteplase 40 mg intravenous push.

A

[self Prevention] A 63-y/o white man (72 inches, 100 kg) with NKDA presents to the medication management clinic for a CV risk reduction assessment. His blood pressure is 152/94 mm Hg averaged over 2 separate readings and HR is 72 bpm. Laboratory tests show potassium (K) 4.3 mEq/L, SCr 0.89 mg/dL, hemoglobin A1C 6.7% (rechecked value: 6.8%), total cholesterol (TC) 169 mg/dL, HDL 40 mg/dL, and LDL 129 mg/dL. The patient takes lisinopril 10 mg daily for his blood pressure. The patient participates in shared decision-making and wishes to undertake aggressive risk reduction efforts. Which group of additional medications would best be added to this patient's daily regimen? A. Aspirin 81 mg daily, atorvastatin 20 mg daily, metformin 500 mg twice daily. B. Aspirin 81 mg daily, atenolol 50 mg daily, lovastatin 10 mg daily. C. Aspirin 325 mg daily, atorvastatin 40 mg daily. D. Aspirin 325 mg daily, glipizide 5 mg twice daily before meals.

A

[self-ass HTN] R.P. is a 58-year-old Japanese-American woman with HTN and OA. Her medications include acetaminophen 3 g/day, amlodipine 10 mg/day, lisinopril 40 mg/day, and aspirin 81 mg/day. Her vital signs include BP 136/85 mm Hg and HR 86 beats/minute. Her total cholesterol (TC) is 180 mg/dL, high-density lipoprotein cholesterol (HDL) 45 mg/dL, and low- density lipoprotein cholesterol (LDL) 115 mg/dL. Her current 10-year atherosclerotic cardiovascular disease (ASCVD) risk is 14.3%. Which is the next best step for R.P. at this time? A. Add chlorthalidone 12.5 mg/day. B. Increase lisinopril to 80 mg/day. C. Add atenolol 50 mg/day. D. Make no medication changes at this visit.

A

Despite IV loop diuretics for several days (increase 10 lb/2weeks) the patient's condition continues to deteriorate, and neurohormonal antagonists had to be discontinued. His most recent blood pressure was 92/56 mmHg and his heart rate was 86 bpm. The team decides to place a PAC, which reveals the following: PCWP 22 mmHg, CI 1.6 L/min/m2, and SVR 920 dyne·s/cm5. Which of the following is best to add at this time? A. Dobutamine 2.5 mcg/kg/min intravenous infusion. B. Sodium nitroprusside 0.2 mcg/kg/min infusion. C. Dopamine 2 mcg/kg/min infusion. D. Norepinephrine 0.05 mcg/kg/min infusion.

A BP low, CI (2.2) and SVR (900) low - should use inotrope

A patient wants to make a tobacco quit attempt. He reports using 2.5 cans of chewing tobacco weekly, with the first chew 15-30 minutes after awakening most days of the week. Which is most reasonable for use during his quit attempt? A. Nicotine 21-mg patch daily for 6 weeks, then a 14-mg patch daily for 4 weeks, then a 7-mg patch for 4 weeks, in combination with 4-mg nicotine gum up to 10-12 pieces/day for 12 weeks. B. Nicotine 42-mg patches daily for 6 weeks, then a 21-mg patch daily for 4 weeks, then a 14-mg patch for 4 weeks, then a 7-mg patch daily for 4 weeks in combination with 4-mg nicotine gum up to 10-12 pieces/day for 12 weeks. C. Nicotine 14-mg patch daily for 6 weeks, then a 7-mg patch for 4 weeks, in combination with 2-mg nicotine gum up to 10-12 pieces/day for 12 weeks. D. Nicotine 21-mg patch daily for 6 weeks, then a 14-mg patch daily for 4 weeks, then a 7-mg patch for 4 weeks, in combination with 2-mg nicotine gum up to 10-12 pieces/day for 12 weeks.

A 2 cans: 21. 3 cans: 42 mg within 30: 4 mg

Which recommendation for nicotine replacement is most appropriate for a 38-year-old man who wishes to stop smoking after a 10 pack-year history? his first cigarette within 1 hour of awakening each day and uses 8 or 9 cigarettes each day. A. Nicotine 14-mg patches: 1 daily for 6 weeks, then 7-mg patches daily for 2-4 weeks. B. Nicotine 7-mg patches: 1 daily for 6 weeks, then discontinue. C. Nicotine 21-mg patches: 1 daily for 2 weeks, then 14-mg patches daily for 2-4 weeks, then 7 mg patches daily for 2 weeks, then discontinue. D. Nicotine gum 4 mg 1 or 2 pieces every 1-2 hours for 12 weeks, then taper

A < 10 cigarettes: 14

A 42-year-old woman presents to the pharmacy with a desire to quit smoking. Her social history: 2 pack/day smoking, the first cigarette smoked within 15 minutes of awakening. She has not previously tried any smoking cessation products, given that this is her first quit attempt. The patient is also currently without prescription insurance coverage. Which initial smoking cessation treatment is most appropriate at this time to augment the cognitive behavioral support she is receiving from 1-800-QUIT NOW? A. Nicotine lozenge 4 mg 1 piece every 1-2 hours. B. Nicotine 14 mg 24-hour patch daily. C. Nicotine gum 2 mg 1 piece every 1-2 hours. D. Varenicline 0.5 mg orally daily.

A < 30 min: 4

A 68-year-old female patient has a 20 pack-year history and currently smokes 15 cigarettes daily with the first cigarette within 10 minutes of waking daily. She has a medical history of anxiety and is working to taper and discontinue clonazepam 0.5 mg twice daily for GAD. Which smoking cessation plan would be most effective for this patient? A. Varenicline 0.5 mg daily for 3 days; then 0.5 mg twice daily for 4 days; then 1 mg twice daily. B. Nicotine patch 21 mg/day topically and bupro-pion extended release (XL) 150 mg orally t w ice d ai ly. C. Nicotine patch 14 mg/day topically and bupropion XL 150 mg orally twice daily. D. Bupropion XL 150 mg orally twice daily.

A Bupropion: CI in pt discontinue benzo

J.P. is a 72-year-old woman with a history of HTN, HFrEF LVEF 20%, and paroxysmal AF. Her estimated creatinine clearance (CrCl) is 80 m L/m i nute/1.73 m2. She currently takes metoprolol succinate 200 mg orally once daily, furosemide 40 mg orally once daily, sacubitril/valsartan 97/103 mg orally twice daily, and spironolactone 25 mg orally once daily. She continues to have episodes of palpitations and dizziness once or twice weekly, which last for about 4-6 hours. During these episodes, she measures her heart rate, recording heart rates of 95-100 beats/minute. Which is the most appropriate therapy at this time?A. Amiodarone 200 mg orally twice daily for 4 weeks, followed by 200 mg orally once daily. B. Dronedarone 400 mg orally every 12 hours. C. Flecainide 100 mg orally every 12 hours. D. Sotalol 80 mg orally twice daily.

A Dronedarone, Flecanide: CI in HFrEF Sotalol: not for HFrEF on BB

D.F. is an 84-year-old woman with a medical history significant for type 2 DM (diet controlled), OA, hypercholesterolemia, and HTN who presents to the clinic with a 1-year history of SOBed over the past month. Her medications are lisinopril 20 mg daily, alendronate 70 mg once weekly, calcium/vitamin D 500 ng/125 IU three times daily, simvastatin 20 mg daily, and aspirin 81 mg daily. Vital signs include blood pressure 120/80 mm Hg and heart rate 80 beats/minute. Physical examination reveals lungs: + rales/rhonchi; cardiac: JVP elevated to 7 cm with a large V wave, S3 present; and extremities: 2+ edema to knee bilaterally. Laboratory values are as follows: SCr 1.2 mg/dL, K 4.7 mEq/L, and BNP 856 pg/mL. An ECHO reveals an LVEF of 66%, mild tricuspid regurgitation, mild mitral regurgitation, and LV hypertrophy. Which is the most appropriate therapeutic recommendation for D.F.'s HF at this time? A. Add furosemide. B. Add hydrochlorothiazide. C. Add spironolactone. D. Add isosorbide mononitrate.

A HFpEF - fluid overload Spironolactone would not provide appropriate lvl of diuretics

Despite initial attempts at diuresis, the patient is only net negative 1 L over the next 24h. Which of the following would most effectively augment his diuresis? A .Metolazone 5 mg by mouth once daily. B. Hydrochlorothiazide 50 mg by mouth once daily. C. Spironolactone 100 mg by mouth once daily. D. Dopamine 2 mcg/kg/min intravenous infusion.

A Less likely response to HCTZ 2/2 worsening renal function Metolazone similar efficacy to chlorthalidone with longer duration

A 59-y/o man is day 1 after a CAGB procedure. He is in the coronary care unit, where a pulmonary artery catheter is placed. His morning laboratory values have shown about a 3-point decrease in hemoglobin (Hgb) from 10.1 g/dL to 7.1 g/ dL since surgery. He has a consistent blood pressure of around 82/36 mm Hg this morning. His pulmonary artery catheter values are as follows: central venous pressure (CVP) 2 mm Hg, pulmonary capillary wedge pressure (PCWP) 8 mm Hg, cardiac output (CO) 3 L/ minute, and systemic vascular resistance (SVR) 1300 dynes × seconds × cm-5. Bedside echocardiogram reveals collapse of the left ventricular (LV) wall, no right ventricular (RV) strain, ejection fraction 40%-45%, and no tamponade. The patient does not have a fever. Which shock state does this patient's hemodynamic and clinical picture most consistently represent? A. Hypovolemic. B. Obstructive. C. Vasodilatory. D. Cardiogenic.

A Low CVP, PCWP, CO; decrease HgB Obstructive: high CVP, RV strain/tamponade No fever Low CVP, PCWP: do not support cardiogenic shock

In addition to nonpharmacologic recommendations, which treatment option is best for his TG? A. Add fenofibrate 145 mg/day. B. Add niacin extended release 500 mg/night. C. Add omega-3 fatty acids 1 g/daily. D. Increase rosuvastatin to 40 mg/day.

A Niacin: CI in PUD, worsen hyperglycemic Increase statin dose no help

A 40-year-old man presents to the ED with blood pressure 200/100 mm Hg, signs of shortness of breath, and encephalopathy. His urine drug screen is positive for methamphetamine. Which is the most appropriate goal for his blood pressure reduction? A. Goal SBP reduction by 25% during the first 60 minutes. B. Goal SBP reduction by 50% during the first 60 minutes. C. Goal SBP reduction by 25% during the first 24 hours. D. Goal SBP reduction by 50% during the first 24 hours.

A No emergencies (AD, stroke)

A 72-year-old man (67 inches [170 cm], 65 kg) is brought to the ED with stroke symptoms that presented 2 hours before arrival. Door-to-CT completion time was 30 minutes. The CT results, interpreted at the time of the test, revealed a right frontal lobe infarction. His medical history includes DM, MI, AF, and HTN. His home medications include glyburide, sotalol, and warfarin. He took aspirin 325 mg at symptom onset. His international normalized ratio (INR) is 1.4, platelet count (Plt) is 174,000/mm3, prothrombin time is 20 seconds, partial thromboplastin time is 32 seconds, and blood pressure is 168/74 mm Hg; ECG reveals a heart rate of 120 bpm with atrial fibrillation. Other pertinent laboratory results include glucose 97 mg/dL, sodium (Na) 137 mEq/L, K 3.6 mEq/L, blood urea nitrogen (BUN) 15 mg/dL, and SCr 0.5 mg/dL. His National Institutes of Health Stroke Scale (NIHSS) score is 22. Urine drug and tobacco screen were negative. Which best represents the role of aspirin in acute stroke? A. Aspirin at an initial dose of 325 mg should be administered within 24-48 hours from symptom onset. B. Aspirin should be administered on hospital arrival, irrespective of the treatment strategy chosen. C. Aspirin should only be administered if fibrinolytic therapy is not given. D. Aspirin at the initial dose should be administered within 72 hours of symptom onset.

A Not recommend given immediately after arrival Delay 24h after alteplase

A 75-year-old African American woman (86 kg) presents to the ED with chest pressure (10/10). Her echocardiogram reveals nonSTEMI in inferior leads. Her medical history is significant for HTN and CKD. Pertinent laboratory results are troponin 4.8 ng/L, SCr 2.7 mg/dL, and estimated creatinine clearance (CrCl) 24 mL/minute/1.73 m2. She has been given aspirin 325 mg single dose; a nitroglycerin drip, initiated at 5 mcg/minute, will be titrated to chest pain relief and BP. She consents for cardiac catheterization after adequate hydration. Which anticoagulation strategy is most appropriate to initiate in this patient? A. Intravenous heparin 4000-unit intravenous bolus, followed by a 1000-unit/hour continuous infusion. B. Enoxaparin 90 mg subcutaneously every 12 hours. C. Fondaparinux 2.5 mg subcutaneously daily. D. Bivalirudin 64.5-mg bolus, followed by a 151-mg/hour infusion.

A Options: UFH, enox, bival UFH: mx 4000 - 1000 enox and bival require renally adj (SCr: 2.7) No fondaparinux - risk of catheter-related thrombosis - load with heparin or bivalirudin if use

[pt case HF] K.S. is a 76-y/o woman with obesity who presents with mild exertional dyspnea. She becomes SOB when walking fast or on hills but still walks about 1⁄4 mile per day. She also admits having 1 pillow orthopnea and paroxysmal nocturnal dyspnea. She has had mild edema in the evenings for many years. She denies angina, palpitations, or syncope. She is a nonsmoker. She has a history of HTN, MI 2 years ago, depression, GERD, and hyperlipidemia. Her medications consist of metoprolol succinate 150 mg daily, lisinopril 10 mg daily, furosemide 20 mg daily, aspirin 81 mg daily, sertraline 50 mg daily, omeprazole 20 mg daily, and simvastatin 20 mg at qhs. Her vital signs today include BP 178/85 mm Hg and HR 62 beats/minute. Her physical examination is positive for JVP, S3 present; trace edema in both extremities; and lungs with slight crackles. Laboratory findings show SCr 2.5 mg/dL and K 4.5 mEq/L. Her electrocardiogram (ECG) reveals normal sinus rhythm (NSR). Given K.S.'s clinical presentation, which symptom has the greatest sensitivity to detect HF? A. Shortness of breath. B. Paroxysmal nocturnal dyspnea. C. Orthopnea. D. Edema.

A SOB: up to 100% sensitive But nocturnal dyspnea has the most specificity

A 66-year-old woman (weight 70 kg) with a history of MI, hypertension, hyperlipidemia, and diabetes mel-litus presents with sudden-onset diaphoresis, nausea, vomiting, and dyspnea, followed by a bandlike upper chest pain (8/10) radiating to her left arm. She had felt well until 1 month ago, when she noticed her typical angina was occurring with less exertion. Her ECG reveals ST-segment depression in leads II, III, and aVF and hyperdynamic T waves and positive cardiac enzymes. Blood pressure is 150/90 mm Hg, and all laboratory results are normal; SCr is 1.2 mg/dL. Home medications are aspirin 81 mg/day, simvastatin 40 mg every night, metoprolol 50 mg twice daily, and metformin 1 g twice daily. Which regimen is best for this patient? A. Aspirin 325 mg, ticagrelor 180 mg one dose, and UFH 60-unit/kg bolus; then 12 units/kg/hour titrated to 50-70 seconds with an early invasive approach. B. Aspirin 325 mg and enoxaparin 70 mg subcutaneously twice daily with an early invasive approach. C. An ischemia-guided strategy with tirofiban 25 mcg/kg; then 0.15 mg/kg/minute plus enoxaparin 80 mg subcutaneously twice daily, aspirin 325 mg/day, and clopidogrel 300 mg one dose; then 75 mg once daily. D. An ischemia-guided strategy with aspirin 325 mg and ticagrelor 180 mg one dose; plus UFH 70-unit/kg bolus; then 15 units/kg/hour.

A STEMI with multiple risk (hx MI, positive troponin, CAD) - no ischemia-guide approach (medical management)

A 75-year-old man with severe chest pain and positive troponins undergoes coronary angiography, which shows three-vessel disease. He is scheduled for urgent CABG surgery. He had received a DES 6 months earlier and was receiving ticagrelor 90 mg twice daily together with aspirin 81 mg daily. Which is the best antiplatelet management strategy before surgery? A. Discontinue ticagrelor; go ahead with surgery after 24 hours; continue aspirin. B. Recommend that surgery be postponed for 7 days; discontinue ticagrelor and aspirin. C. Recommend that surgery be postponed for 5 days; discontinue ticagrelor and aspirin. D. Go ahead with surgery, and continue both ticagrelor 90 mg twice daily and aspirin

A Urgent: dc tica 24h

A 49-year-old man with obesity and a history of DM, HTN, and gouty arthritis has bilateral calf pain with his daily walk. The pain, which he describes "crampy and achy," is relieved when he sits down to rest. He has a 30 pack-year hx of smoking but quit 6 months ago when he was given a diagnosis of DM. On PE, his feet are cool to the touch, and his dorsalis pedis and posterior tibial pulses are diminished. His BP is 149/82 mm Hg, and laboratory results include A1C 8.5% and LDL 135 mg/dL. His home drugs include metformin1000 mg twice daily, gabapentin 100 mg three times daily, amlodipine 5 mg daily, and colchicine as needed. According to the AHA/ACC guidelines, which would best reduce this patient's risk of cardio- and cerebro-vascular disease? A. Aspirin 81 mg daily. B. Aspirin 81 mg daily and clopidogrel 75 mg daily. C. Ticagrelor 90 mg twice daily. D. Vorapaxar 2.08 mg daily with aspirin 81 mg daily.

A Vorapaxar: net benefit uncertain - IIb recommendation

A 55-year-old man (75 kg) presents to the hospital with the chief concern of chest pain that was unrelieved at home with nitroglycerin. His ECG reveals ST-segment depression and T-wave inversion. Cardiac markers show an elevated troponin I. His medical history is significant for CAD, HTN, and DM. His thrombolysis in myocardial infarction (TIMI) score is 4, and his SCr is 1.0 mg/dL. The patient had a recent admission for ACS about 6 months ago. During his previous hospitalization, the patient was thought to have developed HIT because his Plt dropped to 40,000/mm3 after his previous catheterization. Given this patient's diagnosis and history, and if he is taken to the catheterization laboratory for an early invasive strategy, which treatment regimen would be most appropriate to replace unfractionated heparin during his treatment? A. Bivalirudin 56.2-mg intravenous bolus, fol-lowed by a 131-mg/hour infusion. B. Enoxaparin 80 mg subcutaneously every 12 hours. C. Abciximab 18.74-mg/kg intravenous bolus, followed by 9.4 mcg/kg/minute. D. Fondaparinux 2.5 mg subcutaneously daily

A abciximab: most likely cause thrombocytopenia

D.R. is a 76-year-old man who is hospitalized in the ICU and undergoing IV antibiotic treatment for sepsis. During his ICU stay, his blood pressure suddenly decreases, and his ECG reveals AF. D.R. has a medical history of HTN and HFrEF (left ventricular ejection fraction [LVEF] 35%). His home medications are lisinopril 20 mg orally once daily, carvedilol 12.5 mg orally twice daily, and spironolactone 12.5 mg orally once daily; he is not receiving these medications while in the ICU. His blood pressure during AF is 91/59 mm Hg and heart rate is 147 beats/ minute. Which is the most appropriate treatment? A. Amiodarone 300 mg intravenously over 1 hour. B. Digoxin 0.25 mg intravenously every 4 hours to a maximum dose of 1.5 mg. C. Diltiazem 0.25 mg/kg intravenously, then 5-mg/hour continuous intravenous infusion. D. Esmolol 500 mcg/kg intravenously over 1 minute, followed by a 50-mcg/kg/minute continuous intravenous infusion.

A acute ventricular rate control Dil: CI in HF Prefer amio > BB in critically ill

A 58-year-old Hispanic woman with CAD and D2M presents to the clinic with her HBPM readings. She is frustrated because her blood pressure is still not at goal. She takes chlorthalidone 25 mg/day, lisinopril 40 mg/day, amlodipine 10 mg/day, and metoprolol tartrate 25 mg twice daily. She tried terazosin but had to discontinue it because of dizziness. Her bp today is 148/79 mm Hg with repeat 145/81 mm Hg and heart rate is 58 beats/minute. Laboratory test results are as follows: SCr 1.2 mg/dL (CrCl [adjusted body weight] 51.8 m L/minute/1.73 m2), K 3.6 mEq/L, and Na 142 mEq/L. Her BMI is 27.5 kg/m2 and EF is 45%. Which regimen change, if any, would be best for this patient? A. Initiate spironolactone 25 mg/day. B. Discontinue chlorthalidone and start spironolactone 25 mg/day. C. Increase metoprolol to 50 mg twice daily. D. No change in her current regimen is warranted.

A all optimal doses

M.P., a 52-year-old Asian woman, presents to the clinic to discuss her heartburn symptoms. Her primary care physician has been running late, and she was rushed back to her examination room a few minutes ago. Her bp is 144/84 mm Hg. She has no history of HTN, and the only drug she takes is OTC famotidine. Which is the next best action to take for M.P.? A. Recheck her blood pressure after she has been seated quietly for 5 minutes. B. Initiate hydrochlorothiazide 12.5 mg/day. C. Initiate lisinopril 10 mg/day. D. Her blood pressure is not of concern because she does not have HTN.

A at least 2 BP needed

[self-ass Arrhythm] M.A. is a 57-year-old woman who presents to the emergency department (ED) with complaints of dizziness and syncope. She has no pertinent medical history and currently takes no medications. In the ED, her vital signs are blood pressure 98/68 mm Hg, heart rate 39 beats/minute, respiratory rate 16 breaths/minute, and temperature 37°C (98.6°F). Her electrocardiogram (ECG) reveals a regular rhythm. Which is the best treatment? A. Administer atropine 0.5 mg intravenously. B. Administer epinephrine 2-10 mcg/minute intravenously. C. Administer transcutaneous pacing. D. Treatment is not necessary.

A bradycardia - EPI reserved if unresponsive to atropine

You are discussing medical management with a 52-year-old man with HF with HFpEF (EF of 55%) who was recently discharged from the hospital. He has NYHA III symptoms, which are currently being managed with maximal doses of lisinopril, carvedilol, and furosemide. He asks you about starting digoxin. Which information is most appro- priate to give this patient, considering his NYHA functional class and LVEF? A. Digoxin will not be of benefit. B. Digoxin will decrease your risk of all-cause mortality. C. Digoxin will decrease your risk of HF-related hospitalization. D. Digoxin will decrease your risk of HF-related mortality and hospitalization.

A for pEF: no effect

In the ELITE I study, the investigators found a statistically significant difference in the end point of all-cause mortality in older adults with HFrEF receiving losartan versus captopril. However, the ELITE II study, which had the same study design as ELITE I but with a larger patient population, found no difference in all-cause mortality between those receiving losartan and those receiving captopril. Which best depicts the statistical error made by the ELITE I investigators? A. Type I. B. Type II. C. Selection bias. D. Reporting bias.

A found difference when not (reject null when should accept)

In addition, which other pharmacologic modifications would be best to recommend at the same time to improve this patient's CV risk? A. Increase atorvastatin to 80 mg daily and add lisinopril 5 mg daily. B. Add ranolazine 500 mg twice daily and clopidogrel 75 mg daily. C. Discontinue metoprolol and add clopidogrel 75 mg daily. D. Add amlodipine 5 mg daily and ranolazine 500 mg twice daily.

A lisinopril for DM

A 62-year-old man is transferred to the cardiac ICU at your facility after presenting to an outside hospital with ADHF. Despite receiving furosemide 120 mg intravenous once, followed by an infusion at 10 mg/ hour, his symptoms persisted, and his blood pressure began to decrease. Vital signs now include a blood pressure of 92/58 mmHg and heart rate of 88 bpm. Before his arrival on the unit, a PAC was placed, revealing the following: PCWP 26 mmHg, CI 1.6 L/min/m2, and SVR 880 dyne·s/cm5. Laboratory values include Na 132 mEq/L, potassium 5.2 mEq/L, SCr 2.0 mg/dL (up from 1.0 mg/dL at baseline), AST 67 units/mL, ALT 92 units/mL, and serum lactate 3.2 mmol/L. The patient's extremities are cool, but his mental status is appropriate. Which of the following therapies is best to restore tissue perfusion in this patient? A. Dobutamine 3 mcg/kg/min intravenous infusion. B. Milrinone 0.375 mcg/kg/min intravenous infusion. C. Nitroglycerin 50 mcg/min intravenous infusion. D Sodium nitroprusside 0.3 mcg/kg/min intravenous infusion

A low CO, normal SVR - vasodilator not effective Dobutamine: increase contractivity without effecting BP Milnirone: would lower BP

A patient with NYHA III (LVEF < 35%) presents to your family medicine clinic with new-onset gout pain. Which would be the safest recommendation to relieve the patient's pain and swelling? A. Prednisone. B. Oxycodone. C. Indomethacin. D. Ibuprofen.

A oxy only pain

A 65-year-old AA woman with progressing renal insufficiency presents to the clinic. Her bp is 118/66 mm Hg with repeat 116/64 mm Hg. She takes hydrochlorothiazide 25 mg/day, lisinopril 10 mg/day, amlodipine 5 mg/day, and a multivitamin daily. At her previous clinic visit 2 months ago, her SCr was 1.5 mg/dL (CrCl [ideal body weight] 35 mL/minute/1.73 m2). Today, laboratory test results are SCr 2.0 mg/dL (CrCl [ideal body weight] 26.3 mL/minute/1.73 m2), K 3.3 mEq/L, Na 133 mEq/L, and microalbumin/creatinine 65 mg/g. Her BMI is 21.9 kg/m2. Which change, if any, would be best for her medication regimen? A. Discontinue hydrochlorothiazide. B. Discontinue amlodipine. C. Increase lisinopril to 20 mg/day. D. No change in her current regimen is warranted.

A today CrCl < 30

A 51-year-old female patient (68 inches, 89 kg) who has had diabetes since age 46 presents for an annual med review and wellness check today. Her BP 138/72 mm Hg, HR 88 beats/minute, and microalbumin is 19 mg/mmol. A fasting laboratory panel completed today shows TC 186 mg/dL, TG 220 mg/dL, HDL 45 mg/dL, LDL 98 mg/dL, A1C 9.5%, SCr 1.23 mg/dL, and K 4.5 mEq/L. She stopped smoking at age 40. Her current medication profile includes metformin 1000 mg twice daily, rosuvastatin 40 mg daily, and lisinopril 40 mg daily. Which intervention will best improve this patient's A1C and CV risk factors? A. Lose 5-8 kg with lifestyle modification. B. Start an over-the-counter fish oil supplement daily. C. Change rosuvastatin to simvastatin 10 mg daily. D. Start aspirin 81 mg daily.

A - 10%

6 months later, A.M. and her husband are ready to have children. Which is the best therapeutic plan for A.M.? A. No change in therapy is warranted at this time. B. Discontinue ramipril and replace with labetalol. C. Increase chlorthalidone to 50 mg/day. D. Discontinue all antihypertensive therapy

B

A 54-year-old South Asian man with DM, HTN, dyslipidemia, and (GERD). He is a current smoker. His current medications include lisinopril 20 mg/day, amlodipine 10 mg/day, pravastatin 40 mg/day, and omeprazole 20 mg/day. Fasting laboratory results show glucose 109 mg/dL, TC 197 mg/dL, LDL-C 128 mg/dL, HDL-C 37 mg/dL, and TG 166 mg/dL. His non-HDL is 160 mg/dL. According to the 2018 cholesterol management guidelines, which is the best recommendation at this time to achieve his LDL-C goal? A. Add fenofibrate 145 mg/day. B. Change pravastatin to rosuvastatin 20 mg/day. C. Increase pravastatin to 80 mg/day. D. Add ezetimibe 10 mg/day.

B

A 55-year-old man with an unknown medical history comes to your ED with altered mental status. He is placed in a bed when he suddenly becomes unconscious and pulseless. The nursing staff starts CPR. Which statement best describes the most appropriate treatment? A. Survival from cardiac arrest solely depends on medications and advanced airways; CPR should be halted so that airway and line placement can occur. B. The treatable, reversible causes of cardiac arrest should be reviewed and addressed while CPR and lab/diagnostic tests are being performed. C. If the patient is in PEA or asystole, pads should be placed, the patient cleared, and the shock delivered. D. All patients should be given sodium bicarbonate to prevent acidosis.

B

A 64-year-old AA man with stage 4 CKD (eGFR 20 mL/minute/1.73 m2), HTN, and hyperlipidemia is being treated with atorvastatin 40 mg/day. After a recent hospitalization for acute renal failure, he is being referred to begin hemodialysis 3 times a week. Fasting laboratory results show TC 129 mg/dL, LDL-C 64 mg/dL, HDL-C 47 mg/dL, and TG 90 mg/dL. According to the KDIGO guidelines, which is best to address his hyperlipidemia? A. Change atorvastatin to rosuvastatin 40 mg/day. B. Decrease atorvastatin to 20 mg/day. C. Discontinue atorvastatin. D. Increase atorvastatin to 80 mg/day.

B

A 72-year-old woman with documented PAD, DM, and HTN reports to the clinic for a follow-up. She has participated in a structured exercise program with improvement in her claudication symptoms; however, she continues to have symptoms, limiting her quality of life. Her current medications include aspirin 81 mg daily, ramipril 10 mg daily, and insulin glargine 38 units daily. Which would best minimize her claudication symptoms? A. B-complex vitamin. B. Cilostazol. C. Pentoxifylline. D. Warfarin.

B

A patient presents with stable NYHA III HF (LVEF < 30%) and newly diagnosed type 2 DM with an (eGFR) of 46 mL/minute/1.73 m2. Which of the following medications could increase this patient's risk for a major cardiac event? A. Metformin. B. Alogliptin. C. Empagliflozin. D. Liraglutide.

B

During this clinic visit, sacubitril/valsartan is initiated. Which vasoactive substance would most likely be increased in K.S. after therapy initiation? A. N-terminal proBNP (NT-proBNP). B. BNP. C. Angiotensin II (AT II). D. Aldosterone.

B

R.P. is a 65-year-old Hispanic man who is follow-ing up for his periodic checkup. He has a history of stroke 1 year ago but remains ambulatory and living independently. After his stroke, he began monitoring his BP at home. He remains anxious of healthcare settings since his stroke. 3 months ago, his office bp was 134/90 mm Hg, and he was advised to decrease his salt intake. Today, he proudly reports that his HBPM readings are consistently in the low 120s/70s since he modified his diet. However, his clinic blood pressure is 146/88 mm Hg with similar repeat. Which is the next best step for R.P.? A. He has masked HTN; initiate lisinopril. B. He may have white-coat HTN; order ambulatory blood pressure monitoring (ABPM). C. His HBPM skills need improvement; schedule a class for him. D. No action is needed; his HBPM readings are at goal.

B

Which best describes this patient's risk factors for PAD? A. Age, gout, and HTN. B. Diabetes, smoking, and HTN. C. HTN, peripheral neuropathy, and smoking. D. Diabetes, smoking, and peripheral neuropathy.

B

Which medication most likely contributed to this man's current presentation of ACS? A. Doxazosin may cause rebound chest pain. B. Ibuprofen increases the risk of ACS and stroke. C. Aspirin resistance contributes to subsequent ACS. D. Nitroglycerin, given in the ED, can decrease the effectiveness of ticagrelor.

B

[pt ass - Prevention] Which best represents a moderate-intensity statin dose? A. Lovastatin 20 mg daily. B. Pravastatin 40 mg daily. C. Fluvastatin 40 mg daily. D. Simvastatin 5mg daily

B

A B-type natriuretic peptide (BNP) is obtained. Which best depicts what could lead to a lower-than-expected BNP value in K.S.? A. Advanced age of 75 years. B. Obesity. C. Renal insufficiency. D. HTN.

B A, C: cause higher BNP than expected

D.W. is a 50-year-old AA man being discharged from the hospital after NSTEMI. His medical history is significant for HTN. He was taking hydrochlorothiazide 25 mg/day before hospitalization. An echocardiogram before discharge reveals a left ventricular ejection fraction of more than 60%. His vital signs include blood pressure 150/94 mm Hg and heart rate 80 beats/minute. Which is best for managing his HTN? A Dc hydrochlorothiazide and add diltiazem. B. Continue hydrochlorothiazide and add metoprolol. C. Discontinue hydrochlorothiazide and add losartan. D. Continue hydrochlorothiazide and add losartan

B AA not response well to but still use

D.W. is a 43-year-old woman with no history of cardiovascular disease who presents to the ED with palpitations, dizziness, and lightheadedness. She takes no medications except for occasional acetaminophen for headaches. Her ECG reveals supraventricular tachycardia (SVT) (no evidence of preexcitation). Her blood pressure in the ED is 96/75 mm Hg and heart rate is 132 beats/minute. Coughing, carotid sinus massage, and Valsalva maneuver have not effectively terminated her arrhythmia. Which is the most appropriate treatment? A. Immediate DCC. B. Adenosine 6 mg by rapid intravenous infusion. C. Ibutilide 1 mg intravenously over 10 minutes. D. Procainamide 25 mg/minute continuous IV 17-mg/kg loading dose.

B Adenosine: 1st line Ibutilide, procainamide: for WPW, patient not

A 62-year-old man with EF 30% and HTN presents with SOB and lower extremity edema. He admits to a 10-lb weight gain in the past 2 weeks since his primary care physician increased his carvedilol dose from 6.25 mg twice daily to 12.5 mg twice daily to improve blood pressure. Other medications include sacubitril/valsartan 49/51 mg twice daily, eplerenone 50 mg once daily, and furosemide 80 mg orally twice daily. On admission, blood pressure is 118/82 mmHg and heart rate is 88 bpm. Laboratory values include sodium 138 mEq/L, NT-proBNP 4400 pg/mL, and SCr 1.3 mg/dL (baseline). Other pertinent findings include crackles bilaterally at the bases, and 2+ bilateral lower extremity edema. Which of the following is optimal for managing carvedilol at this time? A. Increase dose to 25 mg twice daily. B. Decrease dose to 6.25 mg twice daily. C. Discontinue carvedilol. D. Change carvedilol to metoprolol succinate 100 mg once daily

B Because carvedilol cause ADHF - reduce; no cardiogenic shock

A 50-year-old woman (85 kg) with a hx of HTN and DM presents with backache, fatigue, and unusual shortness of breath. Her ECG reveals ST-segment depression in the anterior leads. Troponin is negative x 2. Her CrCl is estimated by Cockroft-Gault as 80 mL/minute/1.73 m2. She is initiated on aspirin and ticagrelor, with a plan for an ischemia-guided approach. Which is the best anticoagulant strategy for this patient, in addition to aspirin and ticagrelor? A. Bivalirudin 0.75-mg/kg bolus, followed by a 1.75-mg/kg/hour infusion. B. Unfractionated heparin 60-unit/kg intravenous bolus, followed by a 12-unit/kg/hour infusion. C. Fondaparinux 2.5-mg intravenous bolus, followed by 2.5 mg subcutaneously daily. D. Enoxaparin 30-mg intravenous bolus, followed by 40 mg subcutaneously twice daily.

B Bival: not option for med approach No bolus dosing for enox, fonda if NSTEMI D also incorrect dose

M.D. is a 75-year-old man with a history of hypertension and HFrEF (LVEF 25%) who had an episode of SVT caused by AVNRT that was terminated with intravenous drug therapy. His current medications include carvedilol 25 mg twice daily, furosemide 40 mg once daily, lisinopril 40 mg once daily, and spironolactone 25 mg once daily. After consultation with his medical team, he has decided not to undergo catheter ablation and prefers to take oral drug therapy to reduce the likelihood of SVT recurrence. His SCr is 1.0 mg/dL. Which is the most appropriate therapy at this time?A. Diltiazem CD 240 mg orally once daily. B. Dofetilide 500 mcg orally twice daily. C. Flecainide 50 mg orally every 12 hours. D. Propafenone 150 mg orally every 8 hours.

B CCB/C, D: not preferred/CI HFrEF

A 67-year-old man had an NSTEMI and received a DES to his right coronary artery. He is in the cardiology clinic for a follow-up about 12 months post-discharge without complaint. He has diabetes and HTN, both controlled at the current visit. He takes aspirin 81 mg daily, ticagrelor 90 mg twice daily, metoprolol 50 mg twice daily, lisinopril 10 mg daily, and atorvastatin 80 mg. The physician is considering longer-term DAPT. Which best describes the most evidence-based approach to prolonging the duration of DAPT in this man? A. Risk greater than benefit; discontinue DAPT at this visit. B. Risk greater than benefit; discontinue ticagrelor but continue aspirin indefinitely. C. Benefit greater than risk; continue DAPT at current dosing. D. Benefit greater than risk; decrease ticagrelor to 60 mg and continue aspirin

B DAPT < 2 (-1 for 67, +1 for previous NSTEMI, +1 for DM)

[Pt case SIHD] A 65-year-old man with chronic stable angina undergoes PCI and receives a DES in his mid-circumflex artery. His medications before PCI include aspirin 81 mg daily, metoprolol 25 mg twice daily, and atorvas-tatin 20 mg daily. Laboratory values post-heart catheterization are within normal limits; blood pressure is 156/89 mm Hg and heart rate is 78 beats/minute. According to the ACC/AHA guidelines, which antiplatelet regimen is most appropriate for this patient? A. Aspirin 81 mg daily indefinitely and ticagrelor 90 mg twice daily for 6 months. B. Aspirin 81 mg daily indefinitely and clopidogrel 75 mg daily for 6 months. C. Aspirin 325 mg daily indefinitely and ticagrelor 90 mg twice daily for 12 months. D. Aspirin 325 mg daily indefinitely and clopidogrel 75 mg daily for 12 months.

B DEZ: Dual 6 months Bare stent: 1 month

J.M. is a 64-yo man with no history of CV disease who presents to the ED with palpitations, dizziness, and lightheadedness. His ECG reveals SVT with no evidence of preexcitation. His blood pressure in the ED is 102/80 mm Hg and heart rate is 131 beats/minute. Neither cough nor carotid sinus massage terminate his arrhythmia. In addition, J.M.'s SVT is not responsive to adenosine 6 mg intravenously, followed by two doses of adenosine 12 mg intravenously. Which is the most appropriate treatment? A. Digoxin 0.5-mg intravenous bolus; may follow with 0.25 mg intravenously every 6-8 hours up to a maximum cumulative dose of 1 mg over 24 hours. B. Diltiazem 0.25-mg/kg intravenous loading dose, followed by a 10-mg/hour continuous intravenous infusion. C. Ibutilide 1 mg intravenously, administered over 10 minutes, followed 10 minutes later by a second 1-mg intravenous dose, if necessary. D. Procainamide 50 mg/minute intravenous continuous infusion to a total dose of 17 mg/kg, followed by a continuous infusion of 3 mg/minute.

B Digoxin: not preferred for SVT Ibutilide, procainamide: SVT with preexcitation

J.M. has heard about sacubitril/valsartan and wants to try this new drug. Which would best be considered before initiating sacubitril/valsartan? A. Decrease the dose of metoprolol succinate to minimize hypotension. B. Discontinue lisinopril at least 36 hours before beginning sacubitril/valsartan. C. Obtain a baseline BNP before beginning sacubitril/valsartan. D. Increase furosemide to 100 mg to minimize fluid retention

B Do NT-proBNP, not BNP

B.B is a 69-year-old woman (64 inches, 64 kg) withNYHA III HFrEF (EF 30%). Her bp today is 138/88 mm Hg (138/86 mm Hg on repeat) and heart rate is 56 beats/minute. Laboratory test results today are as follows: SCr 1.1 mg/dL (CrCl [adjusted body weight] 44 mL/minute/1.73 m2), K 3.7 mEq/L, and Na 143 mEq/L. She takes carvedilol 25 mg twice daily, enalapril 10 mg twice daily, and furosemide 40 mg twice daily. Her PCP asks you which medication, if any, should be added to her regimen next. Which is the best response? A. No further medications are indicated because her blood pressure is controlled. B. Start spironolactone 12.5 mg/day. C. Discontinue carvedilol and start metoprolol succinate 50 mg/day. D. Start amlodipine 5 mg/day.

B EF < 35%

A 68-year-old white woman with HTN and hyperlipidemia presents with an MI for which she received 2 drug-eluting stents. Her medications include metoprolol succinate 50 mg/day, lisinopril 20 mg/day, furosemide 20 mg/day, atorvastatin 20 mg/day, omeprazole 20 mg/day, aspirin 81 mg/day, and clopidogrel 75 mg/day. She achieved a 35% reduction in her LDL-C, but she has previously not tolerated higher doses of atorvastatin or rosuvastatin because of myalgia. Fasting laboratory results show TC 157 mg/dL, LDL-C 83 mg/dL, HDL-C 48 mg/dL, and TG 132 mg/dL. Which is the best recommendation at this time to further reduce her risk of recurrent events? A. Add alirocumab 75 mg subcutaneously every 2 weeks. B. Add ezetimibe 10 mg/day. C. Add niacin extended release 500 mg/night. D. Continue atorvastatin 20 mg/day.

B EZ preferred if pt < 25% from LDL goal

53 y.o female with asthma, glaucoma came in for HTN emergency. Agent to choose: A. Fenoldopam B. Nicardipine C. Labetalol D. Enalaprilal q6h

B Fenol: CI glaucoma Labe: asthma prefer infusion

K.R. is a 62-year-old man with a history of hypertension, an MI 3 years ago, and paroxysmal AF. His LVEF is 55%. He currently takes hydrochlorothiazide 25 mg once daily, metoprolol tartrate 100 mg twice daily, lisinopril 20 mg once daily, aspirin 81 mg once daily, atorvastatin 20 mg once daily, and warfarin 5 mg once daily (INR 2.2). He continues to have palpitation and dizziness episodes once or twice weekly, which last about 4-6 hours. Which is the most appropriate therapy at this time?A. Amiodarone 400 mg twice daily for 4 weeks; then 200 mg once daily. B. Dronedarone 400 mg every 12 hours. C. Flecainide 100 mg every 12 hours. D. Propafenone extended release 225 mg every 12 hours.

B Flecainide, propafenone" CI if structural heart dz - increased risk of proarrythmia

Which addresses the most appropriate reperfusion for this woman? A. Fibrinolytic therapy. B. An early invasive strategy. C. An ischemia-guided approach. D. An antithrombotic strategy.

B High TIMI - invasive Low TIMI: ischemic, no troponin

Which will best address this patient's ischemic pain while minimizing the risk of complications periprocedurally? A. Intravenous metoprolol 5 mg x 3. B. Nitroglycerin 10 mcg/minute intravenously. C. Morphine 2 mg intravenously as needed. D. Enalaprilat intravenously.

B IV BB: not preferred - risk of cardiogenic shock, already low HR

A 43-year-old white man with HTN and heterozygous familial hypercholesterolemia presents to your lipid clinic for evaluation of statin intolerance. He reports bilateral muscle aches in his legs with atorvastatin (10 mg, 40 mg), rosuvastatin (5 mg), and pravastatin (20 mg). His fasting laboratory results show TC 267 mg/dL, LDL 200 mg/dL, HDL 38 mg/dL, and TG 143 mg/dL. Which is the best recommendation to achieve at least a 50% reduction in LDL? A. Initiate colesevelam 1.875 g every 12 hours. B. Initiate evolocumab 140 mg subcutaneously every 2 weeks. C. Initiate gemfibrozil 600 mg twice daily. D. Initiate fluvastatin extended release (XL) 20 mg/day.

B LDL > 190

A physician on your team asks you to report an adverse drug reaction (ADR) experienced by a patient taking ticagrelor. The patient had severe dyspnea after he was given a 180-mg loading dose. The dyspnea did not resolve within 48 hours and required drug discontinuation. Clopidogrel was initiated as a replacement because the patient received a DES. Which statement best describes The Joint Commission requirements for institu-tional ADR reporting? A. A MedWatch report must be completed that explains the situation in which the ADR occurred. B. Institutions must create their own definition of ADRs that practitioners will know when and how to complete reporting. C. Because dyspnea is a known adverse event of ticagrelor, no reporting is required. D. Only severe or life-threatening ADRs need to be reported.

B MedWatch: for HC professiona, drug has AEs after approved- not mandatory this case

[self-ass CAD] A 62-year-old man with a medical history of hypertension (HTN), diabetes mellitus (DM), and chronic angina presents to his cardiologist for a routine follow-up. He received a DES to the right coronary artery 10 months ago and states he is feeling well and has no complaints. Current medications include aspirin 81 mg daily, ticagrelor 90 mg twice daily, metoprolol 50 mg twice daily, atorvastatin 40 mg daily, metformin 500 mg twice daily, and nitroglycerin 0.4-mg tablets sublingually as needed for chest pain. Blood pressure is 150/88 mm Hg, heart rate is 62 beats/minute, and LDL is 125 mg/dL; other laboratory values are WNL. Which is the most appropriate therapy for this patient? A. Amlodopine 5 mg daily. B. Lisinopril 10 mg daily. C. Ranolazine 500 mg twice daily. D. Metoprolol 100 mg twice daily.

B No complaint of angina, had DM, HTN

Which of the following interventions is best to address this patient's volume status? A. Add tolvaptan 15 mg once daily, titrated to 60 mg/day. B. Administer bolus furosemide and increase infusion to 20 mg/hour. C. Initiate UF at 250 mL/hour of fluid removal. D Hold diuretics and administer intravenous fluids.

B Only mild hypoNa - no tolvaptant PCWP elevated, CI low - renal dysfunction cause hypoV - need high dose furosemide

Which best depicts the preferred treatment strategy for prasugrel given in addition to aspirin for the prevention of thrombotic events in a patient post-PCI? A. 10 mg for a 70-year-old man (55 kg) with a history of stent thrombosis. B. 10 mg for a 62-year-old woman (80 kg) with a history of chronic kidney disease and a CrCl of 50 mL/minute/1.73 m2. C. 5 mg for a 74-year-old man (weight 70 kg) with a history of DM and TIA. D. 5 mg for a 45-year-old woman7 (weight 65 kg) with a history of stroke after a motor vehicle accident.

B Prasugrel 5 if < 60 kg or 75+ (no renal adj) CI if TIA/ICH

A 57-year-old white man presents to a general cardiology clinic reporting angina during his daily walk that resolves with rest. His medical history includes coronary artery disease, HTN, and hyperlipidemia. His medications: aspirin 81 mg/day, lisinopril 20 mg/day, metoprolol succinate 100 mg/day, isosorbide mononitrate 60 mg/day, and simvastatin 40 mg/day (myalgia reported with both atorvastatin and rosuvastatin). The cardiologist decides to add ranolazine 500 mg twice daily. Which is the best recommendation regarding treatment of his hyperlipidemia at this time? A. Continue simvastatin 40 mg/day. B. Decrease simvastatin to 20 mg/day. C. Increase simvastatin to 80 mg/day. D. Change to lovastatin 40 mg/day.

B Ranolazine: CYP3A4 inhibitor

Which best describes a therapeutic benefit of ticagrelor compared with prasugrel? A. Once-daily administration. B. Shorter discontinuation time before elective CABG. C. May be administered with strong inhibitors of 3A4. D. Lower incidence of bleeding.

B Tica: BID, held 5 days (vs 7 days)

C.B. is a 69-year-old woman (66 kg) with paroxysmal AF, hypertension, diabetes, and chronic kidney disease (CrCl 20 mL/minute/ 1.73 m2). Her current medications are metformin extended release 1000 mg orally once daily, lisinopril 20 mg orally once daily, metoprolol tartrate 50 mg orally twice daily, and aspirin 81 mg orally once daily. Because of lack of transportation, C.B. is not expected to be adherent to monthly interna-tional normalized ratio (INR) monitoring. Which is the most appropriate drug therapy for preventing stroke/systemic embolism in C.B.? A. Anticoagulation is not recommended. B. Give dabigatran 75 mg orally twice daily. C. Give edoxaban 60 mg orally once daily. D. Give rivaroxaban 20 mg orally once daily with the evening meal.

B renal adj

[self acute HF] A 76-year-old man with a past medical history for ischemic cardiomyopathy (EF 25%), HTN, and CKD, presents to the cardiac ICU with worsening dyspnea and 8-kg (17.6-lb) weight gain in the past 2 weeks after running out of his home medications. Previously he was taking asa 81 mg once daily, atorvastatin 40 mg once daily, lisinopril 20 mg once daily, metoprolol succinate 100 mg once daily, spironolactone 25 mg once daily, and furosemide 40 mg twice daily. His vital signs include a BP of 118/68 mmHg and HR of 76 bpm. Lab values include Na 136 mEq/L, K 4.8 mEq/L, SCr 1.9 mg/dL (baseline 1.5 mg/dL), NT-proBNP 9600 pg/mL, AST 28 units/L, and ALT 32 units/L. Other pertinent findings include jugular venous pressure (JVP) of 12 cm. In the ED, he received a single bolus dose of intravenous furosemide 80 mg. Which of the following is the most appropriate strategy for rapidly improving his congestive symptoms with furosemide? A. 40 mg intravenous twice daily. B. 80 mg intravenous twice daily. C. 2.5 mg/hour intravenous infusion. D. 12.5 mg/hour intravenous infusion.

B Up dose 2.5 - 100 mg BID or 8 mg/h

After 10 minutes of resuscitative measures, the rhythm changes from pulseless VT to asystole. Which is the most appropriate action to manage the patient's rhythm disturbance? A. Deliver an unsynchronized shock/defibrillation at 120 J. B. Resume high-quality chest compressions. C. Administer atropine 1 mg intravenously. D. Administer 1 g of magnesium.

B asystole - no more shock - no electrical activity

A 69-year-old woman with a history of MI, diabetes, HTN, and GERD is referred to your lipid clinic because of statin intolerance. She reports myalgias with rosuvastatin and pravastatin (40 mg), liver enzyme elevations with atorvastatin, and GI upset with ezetimibe. Her medications include metformin 1000 mg twice daily, amlodipine 10 mg/day, lisinopril 10 mg/day, and omeprazole 20 mg/day, and she is currently tolerating pravastatin 20 mg/day. Today's fasting laboratory results show TC 238 mg/dL, LDL-C 110 mg/dL, HDL -C 44 mg/dL, and TG 421 mg/dL. Which is the best recommendation at this time to further reduce her ASCVD risk? A. Add colesevelam 1.875 g twice daily. B. Add evolocumab 140 mg subcutaneously every 2 weeks. C. Add fenofibrate 145 mg/day. D. Add omega-3 fatty acids 4 g/day.

B colesevelam: CI in pt with TG > 400

Subset? A. Subset I (warm and dry). B. Subset II (warm and wet). C. Subset III (cold and dry). D. Subset IV (cold and wet).

B dyspnea, normal output (serum lactate, AST/ALT)

In the DOSE trial comparing diuretic strategies in patients with ADHF, patients received the treatment to which they were randomized for the first48 hours. From 48 to 72 hours, clinicians were permitted to make adjustments (e.g., changing dose, adding thiazidetype diuretics) before measurement of end points at 72 hours; some statistically significant differences emerged between the treatment arms. For example, patients in the bolus arm were twice as likely to receive a dose-increase compared with those in the continuous infusion arm. Which best describes the methodological problem these adjustments introduced to the study design? A. Decreased external validity. B. Decreased internal validity. C. Publication bias. D. Sampling bias.

B internal validity: minimize confounding A: resemble practice

A.S. is a 75-year-old woman who presents to the ED with palpitations, dizziness, and lightheadedness that began about 4 hours ago. Her medical history is significant for hypertension and diabetes. Medications before admission include lisinopril 20 mg daily, hydrochlorothiazide 25 mg daily, and metformin 1000 mg once daily in the evening. Her blood pressure in the ED is 78/52 mm Hg, heart rate is 170 beats/minute, and respiratory rate is 24 breaths/minute. On arrival at the ED, she begins to lose consciousness. Her ECG reveals AF. Which is the mostappropriate treatment? A. Amiodarone 300 mg intravenously administered over 1 hour. B. Immediate DCC. C. Digoxin 0.25-mg intravenous loading dose every 4 hours up to 1.5 mg total dose D. Diltiazem 0.25 mg/kg intravenously administered over 2 minutes.

B loss conciousness

A 60-year-old man (weight 75 kg) presents to the ED with crushing substernal chest pain and ST-segment elevations on ECG. He has a medical history of diabetes and a 40 pack-year history of smoking. He is taken immediately to the catheter-ization laboratory for primary PCI, and a drug- eluting stent is placed in his left anterior descend-ing artery. In addition to aspirin, which regimen would best maintain this patient's stent patency? A. Clopidogrel 300-mg LD, followed by 75 mg daily for 12 months. B. Prasugrel 60-mg LD, followed by 10 mg daily for 12 months. C. Ticagrelor 180-mg LD, followed by 90 mg daily for 6 months D. Clopidogrel 600-mg LD, followed by 75 mg daily for 6 months.

B prasugrel prefers over clopidogrel in STEMI

[pt case Dislipidemia] A 47-year-old African American woman who self-initiated lifestyle therapies for the past 12 months presents for further treatment. She has longstanding HTN and asthma and was recently given a diagnosis of diabetes. Her medications are albuterol metered inhaler, lisinopril, amlodipine, and metformin. Her vital signs include blood pressure 126/88 mm Hg. Her laboratory results are as follows: TC 184 mg/dL, LDL-C 107 mg/dL, HDL-C 55 mg/dL, and TC 112 mg/dL. Her PCEs estimate a 10-year ASCVD risk of 6.0%. According to the 2018 cholesterol management guideline, which is the most appropriate next step for this patient? A. Initiate a low-intensity statin because her 10-year risk is less than 7.5%. B. Initiate a moderate-intensity statin because she has diabetes and no diabetes-specific risk enhancers. C. Initiate a high-intensity statin because she has diabetes and at least one diabetes-specific risk enhancer. D. Continue lifestyle therapies and do not initiate statin therapy.

B risk < 7.5% - moderate intensity

A 65-year-old white man with a recent discharge diagnosis of MI presents for a 1-month checkup. He also has HTN. He was discharged home on aspirin 81 mg daily, clopidogrel 75 mg daily, lisinopril 10 mg daily, metoprolol 25 mg twice daily, and atorvastatin 80 mg daily. He reports adherence to his medications, yet he has no medical insurance and prefers generic medications, when possible. Today, his fasting laboratory results include glucose 90 mg/dL, total cholesterol 270 mg/dL, high-density lipoprotein cholesterol (HDL) 31 mg/dL, low-density lipoprotein choles-terol (LDL) 189 mg/dL, and triglycerides (TG) 250 mg/dL. His blood pressure is 166/90 mm Hg. Which is the most appropriate addition in medica-tion therapy for this patient, according to the 2018 American College of Cardiology/American Heart Association Lipid Guidelines? A. Non-statin therapy is not recommended. B. Give ezetimibe 10 mg daily. C. Give fenofibrate 145 mg daily. D. Give evolocumab 420 subcutaneously, monthly.

B LDL not goal 70

[pt case - acute HF] A 70-year-old woman with a history of EF 20% presents with worsening dyspnea over the past several days. About 1 week ago, she was discharged from another facility after being newly diagnosed with A.fib. To control her heart rate, she was started on digoxin 125 mcg once daily and her metoprolol succinate was increased from 50 mg to 150 mg once daily. Her other medications include lisinopril 10 mg once daily, spironolactone 25 mg once daily, torsemide 40 mg once daily, and apixaban 5 mg twice daily. Vital signs include a blood pressure of 118/78 mmHg and a heart rate of 66 bpm. Laboratory values include sodium 138 mEq/L, potassium 4.2 mEq/L, SCr 1.4 mg/dL (baseline), AST 36 units/L, ALT 28 units/L, lactate 1.6 mmol/L, NT-proBNP 4200 pg/mL, and serum digoxin concentration of 0.9 ng/mL. Other findings include an S3 heart sound and crackles bilaterally to one-third the height of the lung fields. Which of the following is the most likely precipitating factor for ADHF in this patient? A. Uncontrolled atrial fibrillation. B. Increased β-blocker dose. C. Nonadherence to diuretic therapy. D. Digoxin toxicity.

B - BB should not be double sooner than 2 weeks HR 66 - ok AF

A 54-year-old man presents to the clinic for a follow-up of his HTN drug regimen. His medical history is significant for HTN and (CAD). To control his blood pressure, he takes amlodipine 10 mg/day, lisinopril 40 mg/day, and chlorthalidone 25 mg/day. Today in the clinic, his VS include BP 154/96 mm Hg and heart rate 55 beats/minute, and his laboratory results are all within normal values. Which would best manage this patient's HTN? A. Change chlorthalidone to hydrochlorothiazide. B. Increase amlodipine to 15 mg/day. C. Add spironolactone 25 mg/day. D. Discontinue lisinopril and start losartan 25 mg /d ay.

C

Despite being net-negative 2 L over the next 12 hours, her symptoms do not improve, and the team is considering adding a thiazide-type diuretic. Which of the following is the greatest advantage of oral metolazone over intravenous chlorothiazide in this setting? A. Greater efficacy. B. Greater safety. C. Longer duration of action. D. Faster onset of action.

C

Discharge plan for 57 y/o ADHF (30%) with allergy to lisinopril (angioedema), BP 112/64, HR 88 A. Continue IV diuresis B Metoprolol succinate at target dose C. Empa 10 mg D. Sacubitril/valsartan 24/26

C

Which best depicts the lifestyle intervention for reducing blood pressure that would be expected to provide greatest effect? A. Salt restriction to 2400 mg/day intake. B. Physical activity 25 min/day most days of the week. C. Intentional weight loss of 10 kg. D. Calcium supplementation with vitamin D.

C

Which of the following best describes a metric used by the AHA's Get with the Guidelines-Heart Failure program to recognize the quality of care provided to patients with heart failure with reduced ejection fraction? A. Receiving an oral diuretic and rescue plan instructions at discharge. B. Having a documented right heart catheterization during hospitalization. C. Receiving an evidence-based β-blocker at discharge. D. Enrollment in an interprofessional disease state management program.

C

In the (CONSENSUS) trial, the original planned sample size was 400 patients with HFrEF who were randomized to receive enalapril or placebo. The population was followed for 6 months and evaluated for time to death. This sample size provided a power of 90% for superiority to reject the null hypothesis. Which β value would best correspond to 90% power? A. 0.01 B. 0.05 C. 0.1 D. 0.2

C 1-beta

In the TACTICS-HF trial, a total of 257 patient with ADHF were randomized within 24 hours to tolvaptan 30 mg/day (n=129) or placebo (n=128). At 72 hours, renal dysfunction was observed in 39 patients randomized to tolvaptan and 27 randomzed to placebo (p=0.037). Which of the following best depicts how many patients on average needed to be exposed to tolvaptan to produce one case of renal dysfunction? A. 1. B. 9. C. 11. D. 14.

C 1/ (39/129 - 27/128)

A 78-year-old white man with HTN presents for a (CV) risk assessment. His medications include lisinopril 20 mg/day, chlorthalidone 12.5 mg/day, and aspirin 81 mg/day. Fasting laboratory results show A1C 5.3%, TC 203 mg/dL, LDL-C 136 mg/dL, HDL-C 44 mg/dL, and TG 115 mg/dL. His vital signs include bp 134/76 mm Hg, and his BMI is 27 kg/m2. His CAC is 120. He is a nonsmoker and drinks 1 glass of red wine nightly. His calculated 10-year ASCVD risk score is 36.9%. Which is the best recommendation at this time regarding the use of statin therapy in this patient? A. Consider additional risk markers (e.g., hs-CRP) before recommending statin therapy. B. Discuss with the patient the potential risk- benefit of statin therapy and consider a low-intensity statin, given his age. C. Initiate a moderate- or high-intensity statin after further risk discussion. D. Initiate high-intensity statin therapy because his CAC is greater than 100.

C 10 year risk: irrelevant because already have ASCVD (HTN?)

A 45-year-old patient underwent an elective percutaneous transluminal coronary angioplasty and drug-eluting stent placement in her right coronary artery. Which best represents the minimum time DAPT should be continued? A. 1 month.B. 3 months.C. 6 months.D. 12 month

C 6 months for elevtives

For which patient would eplerenone 25 mg best be part of the drug treatment regimen for ACS at discharge? A. 48-year-old woman with K 4.8 mg/dL and SCr 2.1 mg/dL already receiving lisinopril and metoprolol and unknown ejection fraction. B. 62-year-old man with diabetes and blood pressure 100/70 mm Hg who cannot tolerate lisinopril with ejection fraction of 45%. C. Patient with ejection fraction of 25% and symptomatic diabetes; tolerating lisinopril and metoprolol with blood pressure 125/85 mm Hg. D. Patient with ejection fraction of 40% not taking metoprolol or lisinopril; blood pressure 140/85 mm Hg.

C Add eple/spi w/in 7 days post-MI already on BB and ACE + either HF sx or DM A not best 2/2 SC > 2

A patient with a body mass index (BMI) of 32 kg/m2 has discussed the role of various obesity man-agement strategies with her provider. She has no clinically significant concomitant disease states aside from a history of borderline hypertension (HTN) and hypothyroidism. According to your clinical assessment, which is most appropriate for this patient to use for weight loss? A. Phentermine. B. Orlistat. C. Naltrexone/bupropion. D. Bariatric surgery

C Avoid phen if anxiety (ok if depression) Orlistat: avoid due to nerrow therapeutic index with levo Bariatric: > 35 (risk) or 40

A 64-year-old woman with SIHD, HTN, HLD, and CKD presents with complaints of angina after walking 1 block. Her medications include aspirin 81 mg daily, amlodipine 10 mg daily, metoprolol succinate 50 mg daily, and rosuvastatin 20 mg daily. Her heart rate is 58 beats/minute and bp is 100/60 mm Hg. Pertinent laboratory values include K 4.1 mEq/L, SCr 1.4 mg/dL, and CrCl 52 mL/minute/1.73 m2. Which is the most appropriate recommendation to treat this patient's angina? A. Increase metoprolol succinate to 100 mg daily. B. Change amlodipine to diltiazem. C. Add ranolazine 500 mg twice daily. D. Add lisinopril 5 mg daily.

C BP too low for BB, CCB

Which is the best recommendation regarding further identification of risk-enhancing factors and need for statin therapy? A. No further evaluation is warranted, given the patient's "low risk." B. No further evaluation is warranted, given the patient's "borderline risk." C. Evaluate for risk-enhancing factors, given the patient's "intermediate risk." D. Obtain a CAC to determine whether statin therapy is indicated.

C CAC only when risk unclear

[pt case Arrythmias] E.B. is a 60-year-old man with a history of psychosis who is admitted to the ED 6 hours after taking 25 tablets of extended-release diltiazem 240 mg. His medical history is significant for hypertension, for which he was receiving metoprolol 100 mg orally twice daily, diltiazem 240 mg once daily, and hydrochlorothiazide 25 mg orally once daily. In the ED, his blood pressure is 68/58 mm Hg and heart rate is 48 beats/minute. His ECG reveals sinus bradycardia. His blood glucose concentration is 160 mg/dL and serum potassium concentration is 3.5 mEq/L. Which is the optimal treatment strategy for E.B.? A. Aminophylline 6 mg/kg administered over 20-30 minutes B. Atropine 0.5 mg intravenously; repeat every 5 minutes to a maximum dose of 3 mg. C. Insulin 1-unit/kg IV bolus followed by 1 international unit/kg/hour, and IV dextrose 25 g followed by 0.5 g/kg/hour. D. Placement of a temporary transvenous pacemaker.

C CCB OD Aminophylline: after heart transplant or spinal cord injury

Because of the patient's poor response, the team places a PAC, which reveals the following: PCWP 30 mmHg, cardiac index (CI) 1.8 L/min/m2, and systemic vascular resistance (SVR) 2000 dyne·s/cm5. His blood pressure is 102/60 mmHg. Which of the following is the most appropriate therapy to add? A. Milrinone 0.5 mcg/kg/min intravenous infusion. B. Dobutamine 3 mcg/kg/min intravenous infusion. C. Sodium nitroprusside 0.2 mcg/kg/min. D. Nitroglycerin 25 mcg/min infusion.

C CI low, but BO still ok - likely result of high SVR - need to reduce afterload (and preload?)

A 55-year-old white man (weight 98 kg) is in the ED with substernal chest pain that radiates to his left jaw. On PE, he has obesity and is in moderate distress. His vital signs include BP 156/90 mm Hg and HR 82 beats/minute. His ECG reveals normal sinus rhythm with ST-segment segment depression in leads II, III, and aVF. His medical history is significant for CAD, diabetes, and active peptic ulcer disease on omeprazole 20 mg twice daily. He had a GI bleed 3 months earlier. He is given asa 325 mg orally x 1, and troponin concentrations are obtained, the first measuring 7.8 ng/mL. His SCr is normal. The interventional cardiologist finds a 90% lesion in the RCA and places a DES. Which is the preferred antiplatelet therapy to be added to aspirin 81 mg daily at this time?A. Cangrelor 30-mcg/kg intravenous bolus, followed by a 4-mcg/kg/minute infusion. B. Prasugrel 60-mg loading dose, followed by 10 mg once daily. C. Ticagrelor 180-mg loading dose, followed by 90 mg twice daily. D. Clopidogrel 600-mg loading dose, followed by 75 mg daily.

C Can tolerate PO - no cangrelor No prasugrel: high risk of bleeding Asa need to reduce to avoid DDI

G.M. is a 59-year-old woman (75 kg) with a history of HTN who presents to the ED with palpitations, dizziness, and lightheadedness that began within the past 12 hours. A recent echocardiogram revealed an LVEF of 55%. Her current medications include lisinopril 20 mg orally once daily and hydro-chlorothiazide 25 mg orally once daily. Her ECG reveals AF with a QRS duration of 80 milliseconds and a QTc interval of 420 milliseconds. Her blood pressure in the ED is 95/65 mm Hg and heart rate is 144 beats/minute. She has eaten two meals today. Which is the most appropriate treatment for conversion of her AF to sinus rhythm? A. Sedate for immediate DCC B. Administer dronedarone 400 mg orally BID .C. Administer ibutilide 1 mg intravenously over 10 minutes. D. Administer sotalol 80 mg orally twice daily.

C Can't be sedate for DCC since eating 2 meals Dronedarone, sotalol: not for conversion Ibutilide: ok if EF > 30%

A 62-year-old man with a history of EF 25% presents to the cardiac ICU with ADHF, and initial efforts to improve his symptoms are ineffective. The team decides to place a PAC and the following values are obtained: PCWP 28 mmHg, CI 1.8 L/min/m2, and SVR 1800 dyne·s/cm5. Based on these findings, which of the following is most likely to improve tissue perfusion? A. Decreasing preload. B. Increasing preload. C. Decreasing afterload. D. Increasing afterload.

C Decrease preload would decrease overload (congestion) but not perfusion (after load)

J.M. is a 62-year-old white man with an MI 3 years ago, HTN, depression, PAD, OA, hypothyroidism, type 2 DM (diet controlled), and HFrEF (LVEF of 25%). His medications include aspirin 81 mg/day, simvastatin 40 mg every night, lisinopril 20 daily, metoprolol succinate 150 mg/day, furosemide 80 mg twice daily, cilostazol 100 mg twice daily, acetaminophen 650 mg four times daily, and levothyroxine 0.1 mg/day. He has no known drug allergies. His vital signs include blood pressure 155/90 mm Hg and HR 65 beats/minute. He weighs 100 kg. Pertinent laboratory results include K 4.1 mEq/L, SCr 1.6 mg/dL, and TSH 2.6 mIU/L. His HF is stable and considered NYHA EF III. Which would best maximize the management of J.M.'s HF? A. Add digoxin 0.125 mg daily. B. Add losartan 25 mg daily. C. Add spironolactone 25 mg daily. D. Add amlodipine 5 mg daily.

C EF < 35%, SCr<2.5, K < 5

A 65-year-old man with chronic stable angina received a DES in his mid-circumflex artery 2 years ago. His med-ical history includes HTN, HLD, SIHD, and DM. His medications include aspirin 81 mg daily, metoprolol 25 mg twice daily, metformin 1000 mg twice daily, and atorvastatin 20 mg daily. Blood pressure is 156/89 mm Hg and heart rate is 90 beats/minute. He states that he continues to have one or two episodes of angina per week when climbing more than two flights of stairs. Pertinent laboratory results are potassium (K) 4.2 mEq/L, SCr 1.0 mg/dL, LDL 105 mg/dL, and A1C 7.0%; other laboratory values are within normal limits.2. Which is the most appropriate adjustment to this patient's medical regimen to improve his angina symptoms? A. Add amlodipine 5 mg daily. B. Add sublingual nitroglycerin as needed. C. Increase metoprolol to 50 mg twice daily. D. Increase atorvastatin to 40 mg daily.

C Goal HR: 55, BP 130/80

Which best describes K.S.'s stage of HF? A. A. B. B. C. C. D. D.

C HFrEF D would be having sx at rest despite maximally tolerated GDMT

Which patient is the best candidate for initiation of ivabradine? A. 55-year-old man with NYHA I HF (LVEF of 60%) and HR 75 bpm taking lisinopril 20 mg daily for HTN. B. 45-year-old man with NYHA II HF (LVEF of 35%) and HR 62 bpm taking lisinopril 40 mg daily, metoprolol succinate 150 mg daily, and spironolactone 25 mg daily. C. 65-year-old woman with NYHA functional class III HF (LVEF of 20%) with heart rate 78 beats/minute taking lisinopril 20 mg daily, eplerenone 50 mg daily, and carvedilol 25 mg twice daily. D. 85-year-old man with NYHA function class IV HF (LVEF of 10%) with heart rate 110 beats/minute receiving outpatient dobutamine therapy and awaiting transplantation.

C HR > 78, on max BB, NSR, NYHA II-III (B: HR 63)

A.B. has her VT terminated by drug therapy. Her medical team ascertains that she is at an increased risk of recurrent VT and SCD. Which treatment option is best to reduce her risk of SCD? A. Amiodarone 400 mg orally once daily. B. Sotalol 80 mg orally twice daily. C. Implantation of an ICD. D. Implantation of a permanent pacemaker

C ICD then if not effective, add amiodarone/sotalol

Which statement best describes the role of IV alteplase for this patient? A. He is not a candidate for alteplase because of aspirin therapy. B. He is not a candidate for alteplase because he is taking warfarin. C. He is a candidate for alteplase because of a lack of contraindications. D. He is a candidate for alteplase because he is younger than 75 years.

C INR < 1.7, plt > 100k

Which dose of sacubitril/valsartan would be best for J.M.? A. 12/12.5 mg twice daily. B. 24/26 mg twice daily. C. 49/51 mg twice daily. D. 97/103 mg twice daily.

C Lisino 20 (>10)

Which is the best evidence-based treatment strategy for this patient if stenting occurs in the catheterization laboratory? A. Aspirin 325 mg, clopidogrel 600-mg loading dose, and unfractionated heparin infusion 80-unit/kg bolus, followed by 18 units/kg/hour. B. Aspirin 81 mg, prasugrel 60-mg loading dose, and unfractionated heparin infusion 60-unit/kg bolus, followed by 12 units/kg/hour. C. Aspirin 325 mg, ticagrelor 180-mg loading dose, and unfractionated heparin infusion 60-unit/kg bolus, followed by 12 units/kg/hour. D. Aspirin 81 mg, prasugrel 60-mg loading dose, and bivalirudin 0.75-mg/kg bolus and 1.75-mg/kg/hour infusion.

C Load asa

Which patient would be the best candidate for an ischemia-guided approach? A. 68-year-old man with HTN, DM, and hyperlipidemia with chest pain, T-wave inversion, and daily aspirin use. B. 45-year-old man with ST-segment depression and positive troponin. C. 65-year-old woman with resolved ECG changes and negative troponins. D. 59-year-old woman with recent stent placement and ST-segment elevation.

C Lowest risk 2/2 negative troponin, resolved sx High risk: troponin + ST elevation (B,D)

A patient with NYHA IV HF (LVEF < 40%) currently receives: lisinopril, bisoprolol, furosemide, digoxin, amlodipine, spironolactone, pregabalin, sertraline, glipizide, and colchicine (prn). Which medication would most likely exacerbate this patient's HF? A. Sertraline. B. Amlodipine. C. Pregabalin. D. Colchicine.

C MOA similar to dihydropyridine CCB

[pt case - HTN] A 46-year-old white woman is new to your clinic. While shopping recently, she took her blood pressure at an automatic blood pressure station, which informed her that her bp is "too high." Today, her blood pressure is 144/90 mm Hg, with similar repeat. She takes no prescription medications, though over the counter, she takes an appetite suppressant, St. John's wort, and ibuprofen 400 mg twice daily. For the past week, she has also taken pseudoephedrine 30 mg twice daily for a cold. She drinks "at least" 4 cups of coffee daily, 2 diet sodas/day, and 2 or 3 alcoholic drinks every evening. Although she has no symptoms of illness or of feeling bad, she has difficulty falling asleep at night. Her laboratory test results today are normal. Her BMI is 30.2 kg/m2. Which is the best intervention for this patient's HTN? A. Initiate exercise and ask her to return in 1 month. B. Initiate lisinopril 10 mg/day and ask her to return in 1 month. C. Discontinue her OTC medications (with appropriate alternatives), decrease coffee and soda intake, lower her alcohol intake to 1 drink/day, and ask her to return in 1 month. D. Discontinue her coffee and alcohol intake, initiate lisinopril 10 mg/day, and ask her to return in 1 month

C May not need antiHTN

A 54-year-old white man with allergic rhinitis and HTN presents for an annual wellness visit. His current medications include loratadine 10 mg/day and hydrochlorothiazide 25 mg/day. He does not use tobacco products and reports 1 or 2 alcoholic drinks per week. He exercises 5 times a week (running and cycling) and follows a Mediterranean-style diet. Fasting laboratory results show TC 189 mg/dL, LDL-C 126 mg/dL, HDL-C 45 mg/dL, and TG 90 mg/dL. His calculated 10-year ASCVD risk score is 5.6% but he is very concerned about his risk of heart disease. Which risk-enhancer should you consider at this time?A. Screen for albuminuria. B. Perform ankle-brachial index measurement. C. Evaluate family history of premature ASCVD. D. Obtain a serum calcium concentration.

C No signs of PAD - no evidence of PAD CAC - not serum calcium for risk enhancer

Which is the most appropriate initial treatment strategy for her (pregnant women, soy allergy) blood pressure? A. Enalaprilat 1.25 mg intravenously every 6 hours. B. Clevidipine intravenously at 1 mg/hour. C. Hydralazine 10 mg intravenous bolus every 30 minutes as needed. D. Phentolamine 20 mg intravenously as needed.

C Pregnancy, soy allergy (no clevidipine) phentolamine: not for preg, for catecholamine crisis

A.B. is a 65-year-old woman who was admitted to the cardiac ICU today with palpitations and lightheadedness. Her echocardiogram reveals an LVEF of 30%. A.B. also has a hx of MI, HTN, and dyslipidemia. Her SCr is 1.0 mg/dL, and she weighs 57 kg. While in the cardiac ICU, she has dizziness and palpitations, and her blood pressure is 95/68 mm Hg. Her ECG reveals monomorphic VT at a rate of 125 beats/minute, which lasts longer than 30 seconds and does not terminate on its own. Which is the most appropriate treatment? A. Intravenous amiodarone 150 mg over 10 minutes, followed by 1 mg/minute x 6 hours; then 0.5 mg/min-ute for 18 hours. B. Intravenous lidocaine 1.0-mg/kg intravenous bolus, followed by 0.50- to 0.75-mg/kg intravenous boluses, to a total loading dose of 3 mg/kg; then a continuous intravenous infusion of 2 mg/minute. C. Procainamide 50-mg/minute intravenous continuous infusion to a total dose of 17 mg/kg, followed by a continuous infusion of 3 mg/minute. D. Sotalol 75 mg intravenously every 12 hours

C Procainamide > amiodarone, sotalol > lidocaine

A patient presents to your community pharmacy for refills of lisinopril, furosemide, metoprolol succinate, and digoxin. When you ask how he is feeling, he says that he has been taking his medications as directed and has been eating a low-Na diet. However, over the past week, he has had (SOB) and has gained 3 kg (6 lb). He now has to sleep on 4 pillows so that he does not get SOB. Which would be most appropriate to recommend for this patient? A. Continue to weigh yourself every day and adhere to the low-sodium diet. B. Consider calling your physician this week because your physician may need to adjust your furosemide dose. C. Go to the emergency department for immediate assistance. D. Your HF symptoms seem to be controlled— good job.

C Pt on Red Zone - need immediate assistant: W.gain > 2.3 kg/week (> yellow), severe swelling in feet, unrelieved SOB, need to sleep straight up

[self HF]J.S. is a 55-year-old woman with trastuzumab-induced cardiomyopathy resulting in valvular damage. Her most recent LVEF is 35%. She continues to have SOB, NYHA III. Her medications include lisinopril 20 mg/day, furosemide 40 mg twice daily, carvedilol 12.5 mg twice daily, spironolactone 25 mg/day, and digoxin 0.125 mg/day. She has been stable on these doses for the past month. Her most recent laboratory results include sodium (Na) 140 mEq/L, potassium (K) 4.0 mEq/L, chloride (Cl) 105 mEq/L, bicarbonate 26 mEq/L, blood urea nitrogen (BUN) 12 mg/dL, serum creatinine (SCr) 0.8 mg/dL, glucose 98 mg/dL, calcium 9.0 mg/dL, phosphorus 2.8 mg/dL, magnesium 2.0 mEq/L, and digoxin lvl 0.9 ng/mL. Her VS today include BP 112/70 mm Hg and HR 68 beats/minute. Her lung examination is clear. Which best depicts J.S.'s heart failure (HF) stage? A. A. B. B. C. C. D. D.

C Stage A: no HF, just risk Stage B: HF w/o s&s

A 60-year-old woman with New York Heart Association (NYHA) class IV heart failure (HF) (heart failure with reduced ejection fraction [HFrEF]) is admitted for increased shortness of breath and dyspnea at rest. Her extremities appear. well perfused, but she has 3+ pitting edema in her lower extremities. Her vital signs include blood pressure 125/70 mm Hg, heart rate 92 beats/ minute, and oxygen saturation (Sao2) 89% on 100% facemask. After initiating an intravenous diuretic, which intravenous agent is best to rapidly treat this patient's pulmonary symptoms? A. Dobutamine. B. Milrinone. C. Nitroglycerin. D. Metoprol

C Warm and wet - IV diuretics, then vasodilator

A 62-year-old woman with a history of HTN, HLD, and asthma was initiated on aspirin 325 mg daily for a recent TIA. Within days, she experienced urticaria and itching. Which would be the most appropriate anti-platelet regimen for preventing stroke in this patient? A. Decrease the aspirin dose to 81 mg daily. B. Change to aspirin 25 mg plus extended-release dipyridamole 200 mg twice daily. C. Give clopidogrel 75 mg daily. D. Give warfarin to a target INR of 2.5.

C - allergy to asa

T.J. is a 45-year-old white woman with a hx of type 2 diabetes treated with glipizide 5 mg/day. She presents to the clinic for a routine follow-up of her diabetes. Her vital signs today include BP (average of two readings) 134/84 mm Hg and HR 70 beats/minute. Her laboratory results are as follows: sodium (Na) 140 mEq/L, potassium (K) 4.0 mEq/L, chloride (Cl) 102 mEq/L, bicar-bonate 28 mEq/L, blood urea nitrogen 14 mg/dL, serum creatinine (SCr) 1.0 mg/dL, and 24-hour urine albumin 36 mg/24 hours. At her last visit, her blood pressure was 136/85 mm Hg. Which is best to manage her HTN at this time? A. No changes are needed; her blood pressure is at goal. B. Begin lifestyle modifications and add amlodipine 5 mg/day. C. Begin lifestyle modifications and add lisinopril 5 mg/day. D. Begin lifestyle modifications and add atenolol 25 mg /d ay.

C albuminuria

A 52-year-old woman has a witnessed cardiac arrest in a shopping mall and is resuscitated with an automatic external defibrillator device. On elec-trophysiologic study, she has inducible VT. Which is most appropriate for reducing the secondary incidence of sudden cardiac death (SCD)? A. Propafenone. B. Amiodarone. C. Implantable cardioverter-defibrillator (ICD). D. Metoprolol.

C better than any drugs

BecauseofL.S.'s many hospitalizations, she is enrolled in the CardioMEMS program.Which would be best to recommend regarding anticoagulation for this patient? A. Add warfarin 5 mg daily (titrate to an INR of 2-3). B. Add aspirin 81 mg/day. C. Add both clopidogrel 75 mg/day and aspirin 81 mg/day. D. Add clopidogrel 75 mg/day.

C dual 1 month then asa indefinitely

C.A. is a 75-year-old woman (56 inches, 71 kg) who resides in an assisted living. She is referred to your cardiology clinic for medication therapy management. She is ambulatory, generally alert, and oriented, with no cognitive impairment. Her medical history is significant for NYHA III HFpEF (LVEF of 55%), HTN, COPD, and OA. She takes the following medications: amlodipine 20 mg daily, furosemide 20 mg daily, fluticasone/salmeterol 45/21 mcg 2 puffs twice daily, albuterol metered dose inhaler as needed, and acetaminophen 500 mg three times daily as needed. On physical examination, C.A. appears well nourished and groomed. She is mildly SOB on exertion but in no apparent pain or distress. Evaluation of her lungs reveals diminished breath sounds in the bases with no adventitious sounds. Abdomen palpation is soft and nontender, with active bowel sounds and no signs of hepatosplenomegaly. She has 1+ nonpitting chronic edema and vascular changes to lower extremities. Her vital signs indicate bp 155/85 mm Hg, hr 100 beats/minute, and oxygen saturation 94% on 2 L/minute. Her physical examination is positive for JVP, S3 present; trace edema in both extremities, and lungs with slight crackles. Laboratory tests show SCr 1.3 mg/dL, K 4.0 mEq/L, and BNP 875 pg/mL. Her CBC is significant for hemoglobin 12 g/dL, hct 23%, and mcv 80 fL/cell. Her iron studies completed before clinic show ferritin 50 ng/ mL and transferrin saturation (TSAT) 15%. Her vitamin B12 concentration is 310 pg/mL. Her ECG reveals NSR. Given C.A.'s clinical presentation and laboratory findings, which would be the best recommendation to improve her QoL? A. Ferrous sulfate 325 mg orally three times daily with ascorbic acid. B. Cyanocobalamin 1000 mcg intramuscularly weekly for 4 weeks. C. Outpatient intravenous iron therapy weekly for 4 weeks. D. Darbepoetin alfa 0.75 mcg/kg once every 2 weeks.

C ferritin < 100 or (100-300 and TSAT < 20%) - FE IV only

A 68-year-old woman presents to the cardiologist with complaints of chest pain on exertion. Her medical hx is significant for CAD, HTN, and HLD. Her medications are metoprolol 50 mg twice daily, aspirin 81 mg daily, isosorbide mononitrate 60 mg daily, and atorvastatin 40 mg daily. Her vital signs include HR 60 beats/minute and blood pressure 100/58 mm Hg. Laboratory values are within normal limits. Which would best control her angina? A. Give amlodopine 5 mg daily. B. Increase metoprolol to 100 mg twice daily. C. Give ranolazine 500 mg twice daily. D. Give lisinopril 5 mg daily.

C low BP/HR

A 60-year-old white man with type 2 diabetes is new to your clinic. Today, his bp is 155/78 mm Hg with repeat 151/73 mm Hg, and heart rate is 80 beats/minute. He is intolerant of two different ACEIs because of cough. He takes metformin 850 mg three times daily, glipizide 10 mg twice daily, HCTZ 25 mg/day, and omeprazole as needed. Laboratory test results are as follows: SCr 1.5 mg/dL (CrCl [ideal body weight] 54 mL/minute/1.73 m2), A1C 6.8%, K 4.0 mEq/L, and microalbumin/creatinine 98.2 mg/g. His BMI is 31.6 kg/m2. Which is best to initiate for his elevated blood pressure? A. Spironolactone 12.5 mg/day. B. Amlodipine 2.5 mg/day. C. Losartan 25 mg/day. D. Metoprolol succinate 25 mg/day.

C microalbuminuria

A 76-year-old male smoker (weight 61 kg) has a history of hypertension, benign prostatic hypertrophy, and lower back pain. Three weeks ago, he began to have substernal chest pain with exertion (together with dys-pnea), which radiated to both arms and was associated with nausea and diaphoresis. These episodes have increased in frequency to four or five times daily; they are relieved with rest. He has never had an ECG. Today, he awoke with 7/10 chest pain and went to the ED of a rural community hospital 2 hours later. He was acutely dyspneic and had ongoing pain. Home medications are aspirin 81 mg/day for 2 months, doxazosin 2 mg/day, and ibuprofen 800 mg three times daily. Vital signs include heart rate 42 beats/minute (sinus bradycardia) and blood pressure 104/48 mm Hg. Laboratory results include blood urea nitrogen (BUN) 45 mg/dL, SCr 2.5 mg/dL, and troponin 1.5 ng/L (normal value less than 0.1 ng/L). His ECG reveals a 3-mm ST-segment elevation. Aspirin, ticagrelor, and sublingual nitroglycerin were given in the ED. The nearest hospital with a catheteriza-tion laboratory facility is 2½ hours away. Which regimen is best? A. Give alteplase 15 units intravenously plus enoxaparin 30-mg intravenous bolus. B. Use an ischemia-guided treatment strategy with UFH 4000-unit intravenous bolus, followed by 800 units intravenously per hour. C. Give tenecteplase 35 mg intravenously plus UFH 4000-unit intravenous bolus followed by 800 units intra-venously per hour. D. Transfer the patient to a facility for primary PCI.

C need thrombolytics should not use enox 2/2 renal - no bolus for 75+ Tecneplase prefers over alteplase due to shorter half-life, ease of admin

C.A.'s furosemide dose is increased; however, she needs better control of her blood pressure. Which would be most appropriate to add to amlodipine? A. Carvedilol. B. Bisoprolol C. Spironolactone. D. Metolazone.

C only ACE/aldos for pEF, no BB, CCB; also bronchospastic dz - nonsel inappropriate (if BB warrented: choose biso - better beta sel)

[self emergency] A 69-year-old woman with a medical history of CAD and HFrEF is admitted to a monitored bed for suspected HF exacerbation. Her laboratory values include serum creatinine (SCr) 1.5 mg/dL (baseline 1.0 mg/dL), potassium (K) 4.1 mEq/L, magnesium 1.8 mEq/L, AST 100 IU/L, and (WBC) 13 x 103 cells/mm3. All other laboratory values are unremarkable. While obtaining a medication history, you notice what appears to be monomorphic ventricular tachycardia (VT) on the monitor with heart rates in the 120-130 bpm and (SBP) of 88/60 mm Hg. You alert the emergency response team, and a biphasic defibrillator is brought to the bedside. A synchronized cardioversion is performed with mild sedation. The patient becomes unresponsive after cardioversion and has no pulse. Cardiopulmonary resuscitation (CPR) is initiated immediately with high-quality chest compressions. A peripheral line is in place, and she is immediately ventilated by bag-valve mask. Which of the following is the most appropriate course of action at this time? A. Administer epinephrine 1 mg intravenously x 1. B. Obtain an advanced airway. C. Deliver an unsynchronized shock at 120 J. D. Administer a dose of amiodarone 300 mg intravenously x 1.

C shock again 1st dose of EPI should be given after 1st shock if no ROSC?

A 76-year-old male smoker (62 kg) has a hx of HTN, BPH, and lower back pain. 3 weeks ago, he began to have substernal chest pain with exertion (together with dyspnea), which radiated to both arms and was associated with nausea and diaphoresis. Episodes have increased to 4-5 times daily; they are relieved with rest. He has never had an ECG. Today, he awoke with 7/10 chest pain and went to the ED of a rural community hospital 2 hours later. He was acutely dyspneic and had ongoing pain. Home medications are aspirin 81 mg/day for 2 months, doxazosin 2 mg/day, and ibuprofen 800 mg three times daily. Vital signs include heart rate 42 beats/minute (sinus bradycardia) and blood pressure 104/48 mm Hg. Laboratory results include blood urea nitrogen 45 mg/dL, SCr 2.5 mg/dL, and troponin 1.5 ng/mL (normal value less than 0.1 ng/mL). His ECG reveals a 3-mm ST-segment elevation. Aspirin, ticagrelor, and sublingual nitroglycerin were given in the ED. The nearest hospital with a catheterization laboratory facility is 2½ hours away. Which regimen is best to recommend? A. Give alteplase 15 units intravenously plus enoxaparin 30-mg intravenous bolus. B. Use an ischemia-guided treatment strategy with unfractionated heparin 4000-unit intravenous bolus, followed by 750 units intravenously per hour. C. Give tenecteplase 35 mg intravenously plus unfractionated heparin 4000-unit intravenous bolus, followed by 800 units intravenously per hour. D. Transfer the patient to a facility for primary PCI.

C tecne given with UFH A incorrect 2/2 require intravenous infusion after B, D: incorrect approach

A 51-year-old white man presents with DM, HTN, hypertrig, and a history of acute pancreatitis. He denies using tobacco but drinks 4-6 beers/day. His current medication regimen is as follows: lisinopril 20 mg/day, chlorthalidone 50 mg/day, amlodipine 10 mg/day, rosuvastatin 20 mg/day, metformin 1000 mg twice daily, and insulin glargine 28 units at bedtime. Although he still has obesity (BMI 38 kg/m2), he reports a 5-lb weight loss over the past month by very-low dietary fat and avoidance of carbohydrates, as well as caloric reduction. Fasting laboratory results show A1C 6.9%, TC 157 mg/dL, LDL-C 78 mg/dL (directly measured), HDL-C 38 mg/dL, and TG 1054 mg/dL. Renal and hepatic function remain within normal limits. He shares with you that he is frustrated with taking "all of these pills every day." Which best describes potential secondary causes that may be contributing to his hypertriglyceridemia? A. Alcohol consumption, poorly controlled diabetes, amlodipine. B. Alcohol consumption, rosuvastatin, weight loss. C. Obesity, alcohol consumption, chlorthalidone. D. Obesity, poorly controlled diabetes, metformin

C well control DM chlorthalidone: increase trig

A 56-year-old white woman with a long-standing history of HTN presents with a blood pressure of 210/120 mm Hg and sharp stabbing chest and back pain. A CT done immediately reveals a type A dissection. She is initiated on an esmolol drip to achieve heart rate and blood pressure goals. VS improved to a blood pressure of 100 mm Hg/80 mm Hg and heart rate 59 beats/minute with esmolol. Which other pharmacologic intervention would be most critical in her treatment at this time? A. Initiate 1 L of 0.9% normal saline. B. Initiate a diltiazem 5-mg/hour intravenous infusion. C. Initiate hydromorphone by patient-controlled analgesia 0.1 mg intravenously with a 5-minute lockout. D. No tx

C - for pain in dissection BP goal 100-120, HR: 60, choice: BB or CCB

AA transferred to your hospital with acute chest pain (9/10) radiating to his back. No family history of CV; however, himself has severe uncontrolled (HTN) associated with nonadherence to medication and appointments. He has no history of smoking or illicit drug use. On PE, temperature and oxygen saturation were normal. Heart rate was 80 bpm and RR was 18 bpm. His blood pressure was 188/85 mm Hg. Laboratory results show normal cardiac enzymes, stable SCr at 1.0 mg/dL, negativeD-dimer, and no ST changes on (ECG). (CT) reveals a type A dissection. Which would be the optimal blood pressure and heart rate target for this patient in the acute phase of management? A. Reduce mean arterial pressure (MAP) by 25% in the first 60 minutes and heart rate to less than 60 beats/minute. B. Reduce SBP to less than 185 mm Hg in 30 minutes and heart rate to less than 80 beats/ minute. C. Reduce SBP to less than 120 mm Hg over 20 minutes and heart rate to less than 60 beats/ minute. D. Reduce SBP to less than 140 mm Hg over 60 minutes and heart rate to less than 80 beats/ minute.

C - goal 110-120 within 20 minutes, HR: 60

A 57-year-old African American woman with HTN and dyslipidemia presents to her primary care provider with a concern of increased thirst and urination. Her medications include lisinopril 20 mg/day, amlodipine 10 mg/day, chlorthalidone 12.5 mg/day, and atorvastatin 10 mg/day. Her laboratory results show a hemo-globin A1C (A1C) of 7.7%, TC 184 mg/dL, LDL-C 103 mg/dL, HDL-C 41 mg/dl, and TG 202 mg/dL. Including her new diagnosis of diabetes, her 10-year ASCVD risk score is 22%. Which is the best recommendation at this time? A. Add fenofibrate 145 mg/day. B. Add niacin extended release 500 mg/night. C. Increase atorvastatin to 40 mg/day. D. Continue atorvastatin 10 mg/day.

C?

A 52-year-old male patient asks for recommendations to improve his CV risk through diet interventions. He asks which dietary intervention might be most beneficial from an overall health perspective. Which is the best response? A. Limit all fats to less than 10% of total calories. B. Limit saturated fats to 5%-6% of dietary intake C. Limit all fats to no more than 30% total intake for daily calories. D. Eliminate trans fat intake.

D

A 60-year-old man with stable ischemic heart disease (SIHD), HTN, hyperlipidemia (HLD), and mild depression presents with angina on exertion. His medications include asa 81 mg daily, NTG 0.4 mg sublingually as needed for chest pain, atorvastatin 20 mg daily, citalopram 40 mg daily, and chlorthalidone 25 mg daily. His BP is 158/92 mm Hg and HF is 68 beats/minute. Laboratory values are within normal limits. Which would best treat his angina? A. Lisinopril 10 mg daily. B. Ranolazine 500 mg twice daily. C. Verapamil 120 mg daily D. Carvedilol 6.25 mg twice daily

D

A 72-year-old woman with a history of several TIAs, HTN, and GERD while taking lisinopril, HCTZ, and omeprazole presents to her physician. She finds it difficult to pay for her medications each month. Which is the best antithrombotic regimen to prevent stroke in this patient? A. Warfarin with target international normalized ratio (INR) 2.5. B. Prasugrel 10 mg daily. C. Clopidogrel 75 mg daily. D. Aspirin 81 mg daily

D

A.M. is a 32-year-old woman (66 inches, 70 kg) with type 1 diabetes and HTN. Her BP has been difficult to control, with her SBP over 170 mm Hg at times. Her current medication regimen is as follows: ramipril 10 mg/day, chlorthalidone 25 mg/day, amlodipine 10 mg/day, ethinyl estradiol 20 mcg/norethindrone 1 mg daily (for the past 5 years), and insulin as directed. Her vital signs today include blood pressure 146/82 mm Hg, repeated BP 142/80 mm Hg; HR 82 beats/minute; and BMI 24.5 kg/m2. A.M. would prefer not to take any more drugs, if possible. Which is the best clinical plan for A.M.? A. No change in therapy is warranted at this time. B. Advise weight loss, and recheck her blood pressure in 3 months. C. Change chlorthalidone to hydrochlorothiazide. D. Discuss changing her contraceptive method.

D

Which is the most appropriate initial management strategy for this patient's shock? A. Initiate a norepinephrine drip. B. Administer hydroxyethyl starch. C. Initiate dobutamine. D. Administer lactated Ringer solution.

D

You have been doing chest compressions for about 2 minutes by the time someone arrives with an AED. Which is the most appropriate next step? A. Change providers and continue chest compressions for another 2 minutes. B. Stop compressions to remove clothing, and place AED pads. C. Provide two rescue breaths; then defibrillate the patient. D. Continue CPR, turn on the AED, attach pads, and wait for AED to begin analyzing the rhythm.

D

A male patient with type 2 diabetes (A1C of 7.2%) and seizure disorder is trying lose additional weight to meet his goal weight of BMI 28 kg/m2. He has been working diligently to increase physical activity and demonstrate good adherence to dietary modification recommendations. Currently, the patient's BMI is 33 kg/m2. Which additional therapy would be most appropriate to meet this patient's goals? A. Bariatric surgery. B. Liraglutide. C. Naltrexone/bupropion. D. Metformin.

D Add lira after metformin

Which agent would be best for initial blood pressure management (urgent HTN with positive methamphetamine)? A. Fenoldopam 0.1 mcg/kg/minute intravenous infusion. B. Esmolol 50 mcg/kg/minute intravenous infusion. C. Hydralazine 20 mg intravenous bolus x 1. D. Nitroprusside 0.5-mcg/kg/minute intravenous infusion.

D Avoid BB for substance use disorder Hydralazine: short acting Fenoldopam: mechanism in dopamine - not suitable (because of substance abuse?)

[pt case ED] The man does not respond on initial assessment. He is not breathing, there is no chest movement, and you cannot feel a pulse. Which best describes the most appropriate initial response? A. Open the airway, deliver two rescue breaths, and await emergency medical services arrival. B. Deliver two rescue breaths, followed by 30 chest compressions. C. Ensure the environment is save, and await emergency medical services arrival. D. Start chest compressions at 100-120 beats/minute.

D B wrong because has to do chest compression first then rescue breaths

An 81-year-old woman [NSTEMI] with 4-vessel coronary artery bypass grafting 7 years ago, HTN, and (CKD) presents with dizziness on standing. Last week, she almost fell after getting out of bed. She has tried increasing her fluid and salt intake, but this has not helped. Today, her blood pressure is 138/74 mm Hg and heart rate is 60 beats/minute while sitting; 1 minute later, she stands, and her blood pressure is 115/70 mm Hg and heart rate is 76 beats/minute. She takes metoprolol tartrate 50 mg twice daily, lisinopril 20 mg/day, aspirin 81 mg/day, and atorvastatin 40 mg/day. Which is the next best step in treating this patient's blood pressure? A. Start midodrine 2.5 mg three times daily. B. Discontinue all of her antihypertensive drugs. C. Start droxidopa 100 mg three times daily. D. Lower her metoprolol dose to 25 mg twice dai

D BB no longer indicated after 3 years

A 75-year-old woman admitted for pneumonia has a history of several non-ST-segment elevation myocardial infarctions (NSTEMIs). She had an episode of sustained ventricular tachycardia (VT) during this hospitalization. Her corrected QT (QTc) interval was 380 milliseconds on the telemetry. Her left ventricular ejection fraction (LVEF) was found to be 25%. Her serum potassium and magnesium were 4.6 mEq/L and 2.2 mg/dL, respectively. Which intravenous agent is most appropriate for this patient's ventricular arrhythmias? A. Procainamide. B. Metoprolol. C. Magnesium. D. Amiodarone.

D BB: ok for asymptomatic nonsustained VT and SVT associated with CAD EF < 40% - cant use procainamide

A 50-year-old African American woman with dyslipidemia presents to your clinic for blood pressure assess-ment. When she participated in a health fair the past week, she was told her blood pressure was "too high." She has smoked for the past 35 years but has recently considered quitting. Her blood pressure today an average of 154/94 mm Hg and heart rate is 72 beats/minute. She takes vitamin D supplementation daily, fish oil, and acetaminophen as needed. Laboratory tests show TC 230 mg/dL, HDL 35 mg/dL, LDL 155 mg/dL, SCr 0.9 mg/dL, K 4.0 mEq/L, and Na 141 mEq/L. Her BMI is 26.0 kg/m2. Her 10-year ASCVD risk is 15.2%. Which is the next best action? A. Treat with education on diet and exercise only. B. Treat with education on diet and exercise, and initiate metoprolol succinate 25 mg/day. C. Treat with education on diet and exercise, and initiate chlorthalidone 12.5 mg/day. D. Treat with education on diet and exercise, and initiate chlorthalidone 12.5 mg/day and amlodipine 5 mg/d

D BMI near normal: diet only help little

Which best describes a therapeutic benefit of bivalirudin over unfractionated heparin? A. Less bleeding than with unfractionated heparin alone. B. Can be given without oral antiplatelet. C. Can be given in renal failure. D. No risk of HIT.

D Bival cleared renally

A 64-year-old man with a history of CAD, HTN, and HLD had a coronary artery stent placed 4 years ago. The patient now has chronic stable angina that occurs after walking 2-3 blocks. No lesions are detected on his coronary angiogram that are amenable to further intervention. His heart rate is 58-62 beats/minute and blood pressure is 142/78 mm Hg. Current medications include aspirin 81 mg daily, atorvastatin 40 mg at bedtime, metoprolol 50 mg twice daily, ramipril 10 mg daily, and tadalafil as needed. Which intervention will be of most benefit for his angina? A. Add isosorbide mononitrate 60 mg daily. B. Add diltiazem 180 mg daily. C. Increase metoprolol to 100 mg twice daily. D. Add amlodipine 5 mg daily

D CI: BB, CCB. nitrate (tada)

A 58-year-old woman with a past medical history of EF 55% and hypertension presents to the ED with worsening dyspnea and lower extremity edema over the past several months. At home she takes amlodipine 10 mg once daily, candesartan 16 mg once daily, and furosemide 40 mg twice daily. On admission her vital signs include a blood pressure of 132/84 mmHg and a heart rate of 78 bpm. Laboratory values include sodium 142 mEq/L, potassium 4.4 mEq/L, SCr 1.2 mg/dL (baseline), and BNP 550 pg/mL. Other pertinent findings include crackles on auscultation and 2+ pitting edema bilaterally. Before being transferred from the ED, she received bumetanide 1 mg intravenous once. Which of the following is the most appropriate initial management plan for this patient's ADHF? A. Nitroglycerin 50 mcg/min intravenous infusion. B. Torsemide 100 mg by mouth once daily. C. Bumetanide 4 mg/hour intravenous infusion. D. Furosemide 5 mg/hour intravenous infusion.

D Loop: DOC, caution NTG in pEF Bumetanide would exceed home dose

L.S., an 87-year-old woman, is referred for a 2nd opinion after four hospitalizations for HF in the past 6 months. She has a long history of HTN. She denies having angina or CAD. She describes symptoms consistent with NYHA III. Both an implantable cardioverter-defibrillator (ICD) and a dual-chamber pacemaker set at 65 bpm were placed in the past 6 months; since then, no ventricular arrhythmia has been noted, and no shocks have been delivered. Her medications are lisinopril 5 mg once daily, digoxin 0.125 mg once daily, carvedilol 6.25 mg twice daily, spironolactone 25 mg once daily, furosemide 20 mg once daily, and KCl 10 mEq once daily. Vital signs include bp 152/84 mm Hg and hr 64 beats/ minute, regular. Laboratory values are SCr 1.2 mg/dL and K 4.7 mEq/L. PE reveals lungs: clear to auscultation; cardiac: JVP elevated to 12 cm with a large V wave, S3 present; and extremities: 2+ edema to knee bilaterally. An ECHO reveals LV end-diastolic dimension 6.0 cm, LVEF 38%, global hypokinesis, and LV wall thickness 13 mm. An ECG reveals R-R 857 milliseconds, PR interval 176 milliseconds, QRS duration 104 milliseconds, and QT/corrected QT interval 424/438 milliseconds. Which would be the most appropriate recommendation for L.S.? A. Add metolazone. B. Add ivabradine. C. Consider cardiac resynchronization therapy (CRT). D. Increase lisinopril and furosemide.

D Need higher doses of ACE, diuretics

A 63-year-old white man was admitted to the hospital last week with an STEMI and discharged after stent placement. He now takes prasugrel 10 mg/day, aspirin 81 mg/day, enalapril 2.5 mg/day, and metoprolol succinate 12.5 mg/day. His bp is 105/70 mm Hg with similar repeat, heart rate is 65 beats/minute, and EF is 50%. He has no OH symptoms but asks whether his blood pressure is too low. Which is the best response to the patient's inquiry?A. His blood pressure is too low; discontinue metoprolol succinate. B. His blood pressure is too low; discontinue enalapril. C. Reassure him that his blood pressure should be this low; no changes are needed. D. Reassure him that there is no concern for his blood pressure; no changes are needed.

D No AE of over treamtent

Which would best maximize the management of her HFrEF? A. Increase digoxin to 0.25 mg daily. B. Increase lisinopril to 40 mg daily. C. Increase spironolactone to 50 mg daily. D. Increase carvedilol to 25 mg twice daily.

D No difference in CV with high/low lisinopril doses, also BP low (but ok with BB?)

A provider asks you to describe the data supporting the use of over the counter OMG3 fatty acids for primary prevention of CVD in patients with DM and normal triglyceride values, and to counsel a patient on the most important components regard-ing adverse effects. Which is the best response? A. No randomized controlled trials have assessed the specific end point of omega-3 fatty acids for primary prevention of CVD, and common adverse effects include bleeding dyscrasias and diarrhea. B. Randomized controlled trials assessing this end point have found that consistent use of omega-3 fatty acids provides positive benefits for primary prevention of CVD. Common adverse effects include bleeding dyscrasias and dyspepsia. C. No randomized controlled trials have assessed the use of omega-3 fatty acids for primary prevention of CVD. Primary adverse events include dyspepsia and taste perversion. D. Randomized controlled trials assessing the use of omega-3 fatty acids for primary prevention of CVD have shown no benefit. Primary adverse events include dyspepsia and taste perversion.

D OTC is 1g - no benefit Icosapent ethyl (Vascepa) ok if max statin with TG > 150 (<499) - 2g BID

According to the 2018 cholesterol management guidelines, which is best described as a moderate-intensity statin dose? A. Pravastatin 20 mg/day. B. Lovastatin 20 mg/day. C. Atorvastatin 40 mg/day. D. Rosuvastatin 10 mg/day.

D Pravas 20, Lovas 20: low intensity

A.S. is a 75-year-old man who presents to the ED with palpitations, dizziness, and lightheadedness. His medical history is significant for moderate mitral valve stenosis for 12 years. He currently takes furosemide 20 mg orally once daily and aspirin 81 mg orally once daily. His ECG reveals atrial fibrillation (AF). His blood pressure in the ED is 110/72 mm Hg and heart rate is 140 beats/minute. Which is the most appropriate treatment? A. Immediate direct current cardioversion (DCC). B. Amiodarone 300 mg intravenously over 1 hour. C. Digoxin 0.25 mg intravenously. D. Diltiazem 0.25 mg/kg intravenously over 2 minutes.

D Rate control digoxin: long onset amio: last line

A 34-year-old woman with a history of heterozygous familial hypercholesterolemia recently tested positive for pregnancy. She takes atorvastatin 40 mg/day and ezetimibe 10 mg/day. Which is the best recommendation at this time? A. Continue atorvastatin; discontinue ezetimibe. B. Continue ezetimibe; discontinue atorvastatin. C. Discontinue both atorvastatin and ezetimibe; initiate alirocumab 75 mg subcutaneously every 2 weeks. D. Discontinue both atorvastatin and ezetimibe; initiate colesevelam 1.875 g twice daily.

D Statin and EZ: not for preg No data on PCSK9

A 31-year-old patient presents to the pharmacy with a prescription for bupropion. The patient informs you that she plans to quit smoking. The new prescription is appropriately dosed, and the patient has no CI. Which counseling point is most appropriate for this patient? A. Administer the medication at bedtime because drowsiness is an adverse effect. B. Patients may gain weight while taking this medication. C. Take the medication twice daily for 3 days; then decrease to daily administration. D. Start the medication 1 week before the anticipated quit date.

D bupropion cause insomnia, not dose after 6pm appetite suppression QD x 3 days, then BID

A 75-year-old AA patient develops a GI bleed while on prasugrel and aspirin. The patient's DAPT is held until he is considered stable. The patient had a DES placed 45 days ago after he was hospitalized for an ACS. Which of the following is the best strategy for DAPT in this patient? A. Switch to ticagrelor 60 mg twice daily without load 24 hours after last prasugrel dose. B. Switch to ticagrelor 90 mg twice daily after a 180-mg load 24 hours after last prasugrel dose. C. Switch to clopidogrel 75 mg per day after a 300-mg load 24 hours after last prasugrel dose. D. Switch to clopidogrel 75 mg daily without load 24 hours after last prasugrel dose..

D deescalate: LD not necessary after 30 days, also not recommended after bleeding

[pt case] A 55-year-old African American woman with obesity (weight 120 kg) and a history of ischemic coronary artery disease and diabetes presents to the ED after developing severe back pain accompanied by nausea and vomiting. She describes a similar event earlier in the morning that awoke her from sleep. Electrocardiographic evaluation reveals ST-segment depression in leads V2-V4. Laboratory values include sodium 144 mEq/L, potassium 3.8 mEq/L, SCr 3.2 mg/dL, and glucose 182 mg/dL. Estimated CrCl is 38 mL/minute/1.73 m2. Cardiac troponin is 3.4 ng/mL. Blood pressure is 149/90-153/92 mm Hg with a heart rate averaging 75-80 beats/minute. She takes hydrochlorothiazide 25 mg/day, metformin 850 mg twice daily, and aspirin 81 mg/day. Which best represents this patient's TIMI risk score? A. 2.B. 3.C. 4.D. 5.

D ischemic, 2 angina event/24h, aspirin in 7 days, elevated troponin, ST changes

A 46-year-old African American woman who was recently (3 months ago) hospitalized for acute pancreatitis (TG greater than 2000 mg/dL) is referred to you for management of hypertriglyceridemia. Her other medical history includes HTN, which is well controlled. Her estimated ASCVD risk is 5.1%. Since her hospitalization, she has lost 10 lb by reducing her intake of simple carbohydrates and walking for 30 minutes five times a week. She is concerned because her weight loss has plateaued in recent weeks. She does not use tobacco or alcohol. Her medications include amlodipine 10 mg/day, losartan 100 mg/day, and a multivitamin. Today's fasting laboratory results show TC 210 mg/dL, LDL (not calculated), HDL 39 mg/dL, and TG 1058 mg/dL. Which is the best treatment recommendation at this time? A. Continue diet, exercise, and weight loss only. B. Initiate atorvastatin 40 mg/day. C. Initiate ezetimibe 10 mg/day. D. Initiate fenofibrate 145 mg/day.

D need TG lowering therapy

[self dyslipidemia] A 44-year-old white woman presents to her primary care provider for her annual physical examination. Over the past 6 months, she has had increased fatigue and has gained about 10 lb with no changes in her lifestyle. Her only medication is loratadine 10 mg/day. Her fasting lipid panel shows TC 231 mg/dL, LDL-C 155 mg/dL, HDL-C 54 mg/dL, and TG 112 mg/dL. Her 10-year ASCVD risk score is 1.2%. Which is the best recommendation at this time? A. Initiate atorvastatin 10 mg/day. B. Initiate rosuvastatin 20 mg/day. C. Obtain hs-CRP. D. Obtain a thyroid panel.

D not indicate statin at this time hsCRP - to determine 10 year risk, but risk is low

A 62-year-old man presents to the ED after sev-eral hours of chest discomfort. His ECG reveals a 1- to 2-mm ST-segment elevation with positive troponins. He has also had increasing shortness of breath and lower-extremity swelling over the past 2-3 weeks. His medical history is significant for tobacco use for 40 years, chronic obstructive pul-monary disease, diabetes, and hypertension. His blood pressure is 102/76 mm Hg and heart rate is 111 beats/minute. He has rales in both lungs and 2-3+ pitting edema in his extremities. His echocar-diogram reveals an ejection fraction (EF) of 25%. After primary percutaneous coronary intervention (PCI), he is transferred to the cardiac intensive care unit. Which best describes the acute use of β-blocker therapy in this patient? A. Give 12.5 mg of oral carvedilol within the first 24 hours. B. Give 5 mg of intravenous metoprolol at the bedside. C. Give 50 mg of oral metoprolol succinate at discharge. D. Give no β-blocker at this time.

D pt not euvolemic, should use diuretics first

A 29-year-old woman at 36 weeks' gestation presents to the ED for acute chest pain, blurred vision, headache, and shortness of breath. Her husband denies any illicit drug, alcohol, or cigarette use. Urine toxicology is negative. Initial vital signs are as follows: blood pressure 202/140 mm Hg, heart rate 88 beats/minute, RR 22 bpm, and pain 9/10 (chest pain). Initial laboratory values are as follows: SCr 2 mg/dL (baseline 0.9 mg/dL), AST 608 U/L, ALT 458 U/L, lipase 20 U/L, total bilirubin 1 mg/dL, direct Bil 0.4 mg/ dL, WBC 6 x 103 cells/mm3, Hgb 9 mg/dL, troponin T 1 ng/mL, and D-dimer less than 0.5 mcg/mL. Chest radiography reveals moderate bilateral pleural effusions and no focal consolidations. She takes only folic acid, omeprazole, and a prenatal vitamin. She has a soy allergy. Which criteria best indicate that she is having a hypertensive emergency and describes the optimal initial treatment targets? A.Blood pressure greater than 180/120 mm Hg and reduce to less than 110/80 mm Hg over 4 hours. B. Blood pressure greater than 200/120 mm Hg and reduce by 30% within 2 hours. C. Blood pressure greater than 180/120 mm Hg and reduce by 25% within minutes to hours. D. Blood pressure greater than 180/120 mm Hg and reduce to 140/80 mm Hg during the first hour.

D severe BP>180, organ damage (elevated troponin, AST, SCr..) goal is < 140

A 35-year-old man who presents after right and left coronary angiogram is admitted because of suspected anaphylaxis to contrast. He was given intramuscular epinephrine in the catheterization laboratory before transfer. On arrival at the unit, his blood pressure is 78/42 mm Hg; he was given 1 L of 0.9% sodium chloride, diphenhydramine, famotidine, and methylprednisolone. The patient remained hypotensive and was given an additional 1 L of 0.9% sodium chloride with some response. His MAP is 62 mm Hg. A central venous catheter shows CVP 3 mm Hg, venous oxygen saturation 61%, and lactate concentration 4.2 mmol/L. Post-catheterization Hgb was 10.5 g/dL (pre-catheterization 12.6 mg/dL). Together with fluid resuscitation, which agent would be best to initiate in this patient? A. Packed red blood cells. B. Vasopressin infusion 0.04 units/minute. C. Fresh frozen plasma. D. Norepinephrine 0.03 mcg/kg/minute.

D vasodilatory (distributive) shock: EPI, fluid, NE

A 55-year-old man (100 kg [220 lb]) with a hx of HTN began having stroke-like symptoms 90 minutes ago. His wife gave him 324 mg aspirin and brought him to the ED. His blood pressure on arrival is 175/92 mm Hg. An urgent head CT confirms the diagnosis of acute ischemic stroke. Which is the most appro- priate treatment for the patient at time? A. Give only conservative management because he was given aspirin. B. Administer alteplase 22 mg as an intravenous bolus over 10 minutes, followed by a 68-mg infusion over 60 minutes. C. Give conservative management because of his elevated blood pressure and aspirin administration. D. Administer an alteplase 9-mg intravenous bolus, followed by 81 mg over 60 minutes.

D no CI

Database for ke articles A. International Pharmaceutical Abstracts B. Iowa Drug Informative Service C. Clin-Alert D. Excerpta Medica

D: comprehensive database, may contain data not dound in typical Medline searh

63 y.o HTN, hyperlipidemia, former tobacco use, BMI 28, 10 years risk 13%. Aspirin?

Not indicate - >60 Criteria: < 60 and ASCVD >10%


Related study sets

Indian States & Union Territories and their Capitals

View Set

PED #4 (Chp 43, 48, 49, 50, 55, 57)

View Set

Ultimate Set Introduction to Business Chapter 5

View Set

International Business Law Exam 2

View Set